Está en la página 1de 31

Test 1.

vuelta Distancia
Psiquiatra

Test 1. vuelta Distancia

Psiquiatra
TRASTORNOS NEURTICOS 1) La contaminacin.
2) La duda.
1. Una paciente presenta desde su adolescencia episodios 3) La simetra.
recurrentes de palpitaciones y mareo con sensacin de prdida 4) La soledad.
de estabilidad; ocasionalmente ha sufrido desvanecimientos; 5) La enfermedad.
tras repetidas exploraciones somticas negativas comienza
tratamiento psiquitrico. Cul de los siguientes cuadros es 6. En la etiologa del trastorno obsesivo NO es cierto:
RARO que complique el trastorno que sufre la paciente?
1) Existe una asociacin fuerte entre el trastorno de la Tourette
1) Abuso de sedantes. y el TOC.
2) Trastorno obsesivo. 2) Algunos casos infantiles se relacionan con infecciones
3) Depresin. estreptoccicas.
4) Alcoholismo. 3) Hay una disfuncin de los circuitos corticosubcorticales
5) Ansiedad crnica. frontales.
4) El elemento psicopatolgico ms tpico es la sensacin de
2. Ante un paciente que est en plena crisis de angustia, una de duda.
las siguientes medidas es INEFICAZ: 5) Se ha demostrado una alteracin selectiva de la neurotrans-
misin noradrenrgica.
1) Buspirona.
2) Lorazepam sublingual. 7. Cul de las opciones de tratamiento parece la ms potente
3) Control de la respiracin. para un paciente dominado por la necesidad de comprobar
4) Diazepam oral. cada pequeo acto cotidiano un nmero jo de veces, hecho
5) Explicacin de la benignidad del cuadro. ste que le provoca gran ansiedad, pero que dice ser incapaz
de controlar?
3. Cul de las situaciones que se mencionan es MENOS probable
que desencadene miedo en un agorafbico? 1) Psicoanlisis.
2) Sertralina.
1) Metro. 3) Fenelcina.
2) Centros comerciales. 4) Lorazepam.
3) Hablar en pblico. 5) Clomipramina.
4) Ascensores.
5) Aglomeraciones. 8. Uno de los siguientes sntomas NO forma parte de los criterios
diagnsticos del trastorno por estrs postraumtico:
4. La fobia social es ms DIFCIL de diferenciar de:
1) Pesadillas recurrentes con el suceso traumtico.
1) Trastorno esquizoide. 2) Amnesia de detalles importantes del acontecimiento.
2) Distimia. 3) Estado de nimo depresivo.
3) Personalidad evitativa. 4) Respuesta de sobresalto exagerada.
4) Agorafobia. 5) Evitacin de los lugares relacionados con el suceso.
5) Esquizofrenia residual.
9. La presencia de mltiples sntomas fsicos para los que no se
5. NO es frecuente que los pacientes obsesivos estn preocupados encuentra una causa evidente dene al trastorno conocido
por: como:

CTO Medicina C/Francisco Silvela, 106 28002 - Madrid Tfno. (0034) 91 782 43 30/33/34 E-mail: secretaria@ctomedicina.com www. ctomedicina.com 1
Test 1. vuelta Distancia
Psiquiatra
1) Dismorfofobia. 5) Melancola.
2) Hipocondra.
3) Sndrome de Briquet. 16. El concepto de depresin mayor incluye en la DSM-IV diversos
4) Conversin histrica. subtipos de depresiones, entre los que NO se incluye:
5) Trastorno facticio.
1) Depresin melanclica.
10. En la clasicacin DSM, NO se incluye dentro de los trastornos 2) Depresin psictica.
disociativos: 3) Depresin atpica.
4) Depresin neurtica.
1) Amnesia. 5) Depresin puerperal.
2) Fuga.
3) Personalidad mltiple. 17. Seale el trastorno afectivo que sufre una paciente de 35 aos
4) Despersonalizacin. que desde su adolescencia experimenta de forma casi cons-
5) Convulsiones. tante sntomas depresivos de intensidad leve, asociados con
una sensacin de cansancio enorme y una clara tendencia al
11. Las benzodiacepinas ejercen su accin: sueo, y cuyos sntomas apenas desaparecen unas semanas al
ao, empeorando de forma dramtica cuando la paciente sufre
1) Activando el receptor serotoninrgico 5HT1a. un problema sentimental:
2) Modicando la permeabilidad neuronal al calcio.
3) Potenciando la funcin GABArgica. 1) Depresin mayor.
4) Interriendo con los sistemas de segundo mensajero. 2) Trastorno mixto ansioso-depresivo.
5) Bloqueando la recaptacin noradrenrgica. 3) Personalidad depresiva.
4) Neurastenia.
12. Ante un paciente en el que sospechamos una intoxicacin por 5) Distimia atpica.
alprazolam, el tratamiento de eleccin es:
18. Dentro de los cuadros depresivos del anciano, NO es frecuente
1) Naloxona. que encontremos:
2) Flumacenil.
3) N-acetilcistena. 1) Predominio de los sntomas somticos.
4) Trihexifenidilo. 2) Quejas cognitivas.
5) Fisostigmina. 3) Alto riesgo de suicidio.
4) Frecuente ideacin delirante.
13. En un paciente con hepatopata preferiremos utilizar una de las 5) Mala respuesta al electrochoque.
siguientes benzodiacepinas:
19. Acude a su consulta una mujer de 21 aos que lleva ms de dos
1) Diazepam. aos y medio sintindose cansada, con dicultad para concen-
2) Cloracepato dipotsico. trarse en los estudios. Su familia reere que, durante este tiempo,
3) Clordiacepxido. tambin ha engordado unos 7 kilos debido a que come ms de
4) Alprazolam. lo que lo haca antes. En esta paciente, usted espera encontrar
5) Lorazepam. con ms probabilidad:

14. Cul de las siguientes benzodiacepinas tiene una vida media 1) Insomnio de conciliacin
menor? 2) Despertar precoz
3) Somnolencia excesiva diurna.
1) Midazolam. 4) Despertares nocturnos frecuentes.
2) Flurazepam. 5) Sueo no reparador.
3) Lorazepam.
4) Alprazolam.
5) Clonazepam. 20. Una mujer de mediana edad, sin antecedentes psiquitricos
de inters, que presenta una profunda tristeza desde hace 6
meses as como incapacidad de disfrutar de cosas que antes le
TRASTORNOS AFECTIVOS. FRMACOS ANTIDEPRESIVOS hacan pasrselo bien. Su marido cuenta que ya nada parece
Y ESTABILIZADORES DEL HUMOR hacerle gracia, que come menos y que ha perdido 6 kilos en los
ltimos 3 meses. En la anamnesis, usted esperara encontrar que
15. La forma ms frecuente de trastorno del estado de nimo es, la paciente mejora:
de entre las siguientes:
1) Por la maana.
1) Enfermedad bipolar. 2) Con los IMAO ms que con tricclicos.
2) Distimia. 3) En relacin con factores externos.
3) Trastorno ciclotmico. 4) Al terminar el da.
4) Depresin mayor. 5) En verano.

CTO Medicina C/Francisco Silvela, 106 28002 - Madrid Tfno. (0034) 91 782 43 30/33/34 E-mail: secretaria@ctomedicina.com www. ctomedicina.com 2
Test 1. vuelta Distancia
Psiquiatra
21. La presencia de un sndrome depresivo leve y autolimitado tras 26. Se llama trastorno bipolar-2 a la siguiente combinacin de
la prdida de un ser querido recibe el nombre de: sndromes afectivos:

1) Melancola. 1) Depresin mayor y mana.


2) Depresin reactiva. 2) Depresin mayor sobre una distimia.
3) Duelo no complicado. 3) Depresin mayor e hipomana.
4) Depresin mayor. 4) Mana unipolar (sin depresin asociada).
5) Neurosis depresiva. 5) Depresin invernal y episodios hipertmicos en verano.

22. D su juicio diagnstico en el caso de una mujer de mediana 27. Cul de los siguientes pacientes presenta un mayor riesgo de
edad y sin antecedentes psiquitricos de inters, que presenta suicidio?
profunda tristeza y llanto, estando convencida de padecer una
enfermedad mortal en castigo por los pecados que ha cometido. 1) Varn, 25 aos, con distimia de inicio precoz, sin antecedentes
Ha dejado de tomar alimento alguno, est postrada en la cama de suicidio.
y a duras penas responde a las preguntas que se le hacen: 2) Mujer, 46 aos, con depresin endgena unipolar.
3) Mujer, 85 aos, con depresin reactiva tras su reciente
1) Trastorno distmico. mudanza.
2) Depresin farmacgena. 4) Varn, 72 aos, con melancola delirante.
3) Depresin psictica. 5) Mujer, 19 aos, con depresin bipolar y antecedentes
4) Tristeza postparto. familiares de suicidio.
5) Esquizofrenia catatnica.
28. Las listas de frmacos capaces de causar depresin son muy
23. Un paciente de 62 aos lleva un mes prcticamente sin realizar largas; sin embargo, uno de los siguientes NO se asocia con
ninguna actividad, no tiene ganas ms que de estar en la cama y depresin sino con mana:
dice no tener fuerzas ni para acabar con su vida. Asimismo, est
convencido de estar arruinado a pesar de que, segn su familia, 1) Propranolol.
mantienen una economa desahogada. El personal de enfermera 2) Isoniacida.
ha observado que est algo ms activo por la noche y que coge 3) Neurolpticos.
bien el sueo, pero se despierta a las 3 de la madrugada. Tras 4) Reserpina.
recibir tratamiento antidepresivo con imipramina empieza a 5) Anticonceptivos hormonales.
estar ms inquieto e irritable, hablar durante horas seguidas y
con tendencia a la procacidad y a la desinhibicin sexual. Qu 29. En una de las siguientes indicaciones NO se utilizan antidepre-
debemos considerar en primer lugar? sivos:

1) Que presenta una reaccin psicolgicamente normal tras 1) Trastorno por angustia.
la recuperacin de la depresin. 2) Narcolepsia-cataplejia.
2) Que padece un trastorno bipolar, desconocido hasta ese 3) Dolor crnico neuroptico.
momento. 4) Cefalea tensional.
3) Que el tratamiento antidepresivo est induciendo un cuadro 5) Abstinencia alcohlica.
maniforme.
4) Que estn apareciendo rasgos patolgicos de personalidad, 30. A la hora de seleccionar un antidepresivo para el tratamiento
ocultos por la depresin. de un paciente, cul termina siendo el principal factor impli-
5) Que est simulando, pues esos sntomas no son congruentes cado?
en un trastorno afectivo.
1) Los antecedentes familiares de respuesta a un frmaco.
24. Dentro de las diferencias entre el trastorno bipolar y la depresin 2) El subtipo sintomtico de depresin.
recurrente, NO encontramos: 3) El antecedente de una respuesta personal al frmaco.
4) El perl potencial de efectos adversos.
1) Inicio a una edad ms tarda en el depresivo. 5) Las preferencias del paciente.
2) Ms antecedentes familiares afectivos en el bipolar.
3) Duracin ms larga de las recadas en el depresivo. 31. Una vez superada la fase aguda de una depresin, se reco-
4) Alto riesgo de suicidio en el bipolar. mienda:
5) Equilibrio entre sexos en el depresivo.
1) Retirar el tratamiento antidepresivo.
25. NO sugiere bipolaridad en un paciente depresivo: 2) Reducir la dosis a la mitad y mantener el tratamiento 6
semanas ms.
1) Debut en el puerperio. 3) Mantener al menos 6 meses la misma dosis que produjo la
2) Antecedentes familiares de mana. mejora.
3) Presencia de ideacin delirante incongruente. 4) Dejar una dosis mnima y ecaz para prevenir recadas de
4) Inicio en la adolescencia. forma indenida.
5) Provocacin de hipomanas con antidepresivos. 5) Comentar al paciente que tome el frmaco segn se encuentre.

CTO Medicina C/Francisco Silvela, 106 28002 - Madrid Tfno. (0034) 91 782 43 30/33/34 E-mail: secretaria@ctomedicina.com www. ctomedicina.com 3
Test 1. vuelta Distancia
Psiquiatra
32. Los antidepresivos tricclicos renen las siguientes caracters- 5) En las fases agudas depresivas se recomiendan litemias
ticas, EXCEPTO: inferiores a 1,0 mEq/l.

1) Cardiotoxicidad. 38. Un paciente bipolar en tratamiento con litio ingresa en Urgencias


2) Bajo precio. por un sndrome confusional; al determinar la litemia se obtiene
3) Importantes efectos anticolinrgicos. una cifra de 3,1 mEq/l, cuando haca una semana su litemia era
4) Alta letalidad en sobredosis. de 1,0 mEq/l; suponiendo que el paciente no haya variado la
5) Inicio del tratamiento a dosis teraputicas. dosis prescrita, cul de las siguientes circunstancias explicara
esta cifra?
33. Los ISRS son los antidepresivos ms usados en la actualidad, entre
otras razones porque no producen apenas efectos secundarios 1) Tratamiento concomitante con carbamacepina.
graves; sin embargo, muchos pacientes se van a quejar de: 2) Uso de nifedipino como antihipertensivo.
3) Sobreingesta hdrica.
1) Disfunciones sexuales. 4) Toma de indometacina por un esguince.
2) Somnolencia. 5) Prescripcin de teolina para una enfermedad pulmonar
3) Estreimiento. obstructiva crnica.
4) Poliuria.
5) Sequedad de piel y mucosas. 39. Una paciente de 43 aos, diagnosticada desde los 22 aos de
trastorno bipolar y sin recadas hasta el ltimo ao tomando
34. NO es cierto sobre los frmacos llamados IMAO: exclusivamente litio, durante este ao ha presentado 5 episo-
dios afectivos que su psiquiatra relaciona con la instauracin
1) Los IMAO reversibles (RIMA) han resultado poco ecaces. de paroxetina ante fases depresivas leves y nalmente ha diag-
2) Los IMAO clsicos precisan de una dieta para evitar crisis nosticado a la paciente de cicladora rpida. En estos momentos
serotoninrgicas. est ligeramente deprimida. Cul de las siguientes opciones
3) En la depresin atpica los IMAO obtienen excelentes resul- teraputicas le parece ms adecuada?
tados.
4) Los IMAO selectivos de la MAO-B se usan en la enfermedad 1) Suspender el litio e instaurar el topiramato como base de
de Parkinson. su tratamiento.
5) Est contraindicada su combinacin con ISRS. 2) Mantener el litio y aadir carbamacepina o valproico.
3) Quitar el litio y aadir el valproico y un ISRS, pues la paciente
35. Acerca de la terapia electro-convulsiva (TEC), seale lo FALSO: est deprimida.
4) Mantener el litio y aadir olanzapina.
1) Se puede emplear como primera opcin teraputica en las 5) Quitar el litio y realizar terapia de mantenimiento con TEC.
depresiones con alto riesgo suicida.
2) Sus escasas complicaciones permiten utilizarlo con segu- 40. Uno de los siguientes pacientes NO presenta una mala res-
ridad en embarazadas. puesta al litio:
3) A da de hoy est formalmente contraindicada en cualquier
forma de esquizofrenia. 1) Mana disfrica.
4) Requiere aplicar al paciente anestesia general de corta 2) Recadas estacionales.
duracin y miorrelajacin. 3) Mana secundaria.
5) La hipertensin intracraneal es contraindicacin de este 4) Ciclacin rpida.
tipo de terapias. 5) Fase bipolar mixta.

36. A la hora de iniciar un tratamiento con litio, una de las pruebas


siguientes resulta INNECESARIA: TRASTORNOS PSICTICOS. FRMACOS ANTIPSICTICOS

1) Test de embarazo. 41. Uno de los siguientes datos acerca de la epidemiologa de la


2) Sodio y potasio. esquizofrenia es FALSO:
3) Electrocardiograma.
4) Electroencefalograma. 1) La prevalencia-vida es del 1%.
5) Creatinina y urea. 2) En varones se describe un comienzo ms precoz.
3) Hay una mayor incidencia de enfermedad en clases sociales
37. Una de las siguientes armaciones acerca de las cifras de litemia bajas.
es FALSA: 4) Existe un exceso de esquizofrnicos que han nacido en los
meses fros.
1) Niveles inferiores a 0,4 mEq/l son, en general, inecaces. 5) La esquizofrenia duplica el riesgo de muerte.
2) Niveles superiores a 1,5 mEq/l se asocian con toxicidad.
3) Durante el perodo de mantenimiento suele bastar con 42. El concepto actual de esquizofrenia se apoya en los siguientes
litemias inferiores a 1,0 mEq/l. datos SALVO:
4) En las fases agudas manacas se recomiendan litemias
superiores a 1,0 mEq/l. 1) Curso habitualmente crnico y deteriorante.

CTO Medicina C/Francisco Silvela, 106 28002 - Madrid Tfno. (0034) 91 782 43 30/33/34 E-mail: secretaria@ctomedicina.com www. ctomedicina.com 4
Test 1. vuelta Distancia
Psiquiatra
2) Presencia de crisis alucinatorias y delirantes. 1) Su conciencia de enfermedad.
3) Duracin superior a 6 meses. 2) Los efectos secundarios del tratamiento.
4) Ausencia de factores psicosociales precipitantes. 3) El tipo de esquizofrenia.
5) Cambios afectivos y reduccin de la sociabilidad. 4) El cumplimiento de la medicacin.
5) La emocin expresada en la familia.
43. Cul de los siguientes factores NO se incluye dentro de las
causas de vulnerabilidad a la esquizofrenia? 49. Una paciente le reere que desde hace aos un famoso cantante
le maniesta su amor a travs de insinuaciones o gestos en sus
1) Antecedentes familiares. intervenciones pblicas. Ha tratado, sin xito, de comunicarse
2) Problemas obsttricos. con l mediante llamadas telefnicas, cartas e incluso yendo a
3) Consumo de txicos. su domicilio, por lo que fue denunciada. No sufre alucinaciones
4) Infecciones virales. y su capacidad de juicio, fuera del tema citado, es totalmente
5) Enfermedades infantiles. adecuada. Para distinguir este cuadro de una esquizofrenia,
usted se apoyara en que la enfermedad de la paciente:
44. Un varn de 24 aos es llevado a Urgencias por presentar gran
angustia. Sus padres comentan que llevan notndole raro desde 1) Suele comenzar en edad juvenil.
hace unos 8 meses; ha dejado de salir con sus amigos y de estudiar, 2) No produce deterioro del paciente.
y parece encerrado en s mismo. Los dos ltimos meses tiene una 3) Tiene tendencia a la cronicidad.
mirada rara, segn su madre, quien lo ha encontrado hablando 4) Podra presentar delirios de perjuicio.
slo en numerosas ocasiones. En Urgencias el paciente te comenta 5) Se acompaa de alucinaciones
angustiado que est siendo sometido a una vigilancia y persecucin
asxiante por todas aquellas personas que van en los coches con 50. Un compaero de tu trabajo comienza a sospechar de la delidad
matrcula acabada en M, pues esa letra signica Morirs pronto. de su novia, quien trabaja en otro departamento de la empresa,
Cmo se denomina este ltimo fenmeno psicopatolgico? cercano al tuyo; cada vez que coincids en la cafetera y os ve
hablando, tiene la certeza de que estaris quedando para ms
1) Bloqueo del pensamiento. adelante y si no hablis cree que se debe a que os habis sentido
2) Descarrilamiento. vigilados, cmo se denomina este fenmeno psicopatolgico?
3) Pensamiento disgregado.
4) Percepcin delirante. 1) Formacin reactiva.
5) Idea delirante secundaria. 2) Anulacin.
3) Interpretacin delirante.
45. Seala cul de los siguientes sntomas NO se incluye entre los 4) Aislamiento del afecto.
sntomas de primer rango de la esquizofrenia: 5) Fantasa.

1) Eco del pensamiento. 51. Cul es el mecanismo de accin principal de los antipsicticos
2) Alucinaciones visuales. clsicos?
3) Robo del pensamiento.
4) Vivencias de inuencia. 1) Bloqueo serotoninrgico 5HT1a.
5) Delirios de control. 2) Estmulo GABArgico.
3) Inhibicin de la COMT.
46. Cul de los sntomas siguientes aparece con mayor claridad en 4) Antidopaminrgico D2.
la fase residual de la esquizofrenia? 5) Anticolinrgico muscarnico.

1) Descarrilamientos. 52. Una de las siguientes armaciones sobre los antipsicticos


2) Percepciones delirantes. tpicos es FALSA:
3) Aplanamiento afectivo.
4) Pseudoalucinaciones auditivas. 1) Los antipsicticos tpicos son todos igual de ecaces.
5) Conducta catatnica. 2) La potencia de un antipsictico se reere a su accin anti-
dopaminrgica.
47. El debut precoz de la esquizofrenia se asocia con un peor pro- 3) Cuanto ms potente es un antipsictico ms hipotensin
nstico de la enfermedad, como todos los datos siguientes, produce.
EXCEPTO uno: 4) Los efectos extrapiramidales de los antipsicticos de baja
potencia son escasos.
1) Sexo masculino. 5) La galactorrea se produce por afectacin del sistema tube-
2) Antecedentes familiares de depresin. roinfundibular.
3) Personalidad previa paranoide.
4) Escasa productividad psictica. 53. El mecanismo de accin que parece aumentar la ecacia de la
5) Ausencia de desencadenantes. risperidona y reducir sus efectos extrapiramidales es:

48. La probabilidad de recada en un paciente esquizofrnico NO 1) Bloqueo de la recaptacin de dopamina.


est en relacin con: 2) Antagonismo del receptor H1.

CTO Medicina C/Francisco Silvela, 106 28002 - Madrid Tfno. (0034) 91 782 43 30/33/34 E-mail: secretaria@ctomedicina.com www. ctomedicina.com 5
Test 1. vuelta Distancia
Psiquiatra
3) Potenciacin de la funcin GABArgica. 1) VCM elevado.
4) Bloqueo D2 y antagonismo 5HT2a. 2) GGT mayor de 300.
5) Estimulacin D1 y D4. 3) Descenso de la CDT (desialotransferrina).
4) Hipertermia.
54. Tras una inyeccin de un psicofrmaco un paciente presenta un 5) Convulsiones tnico-clnicas.
movimiento involuntario de los ojos hacia arriba, que le resulta
doloroso y se mantiene a pesar de sus esfuerzos por evitarlo; 59. Un varn de 48 aos con larga historia de dependencia crnica
qu tratamiento precisa? del alcohol, es ingresado en el Servicio de Psiquiatra porque
me persiguen homosexuales que me llaman perverso (sexual).
1) Biperideno. Al pasar por delante de los bares oye comentarios en su interior
2) Amantadina. que dicen que es marica, travestido y que van a ir a por l. Usted
3) Dantroleno. pensar que el paciente presenta una alucinosis alcohlica si
4) Lorazepam. detecta todo lo siguiente, EXCEPTO:
5) Propranolol.
1) El nivel de conciencia del paciente no est alterado.
55. Un paciente esquizofrnico, de 25 aos, lleva 2 semanas tomando 2) Las alucinaciones que presenta son predominantemente
15 mg/da de haloperidol. Los sntomas psicticos han mejorado auditivas.
ostensiblemente. El paciente reere encontrarse muy inquieto, 3) El paciente ha bebido ms de lo habitual durante los
no puede permanecer sentado mucho rato y se tiene que poner a dos ltimos das.
caminar sin rumbo. Cul, entre las siguientes, sera una medida 4) El paciente desarrolla durante el ingreso un delirio ocupa-
INCORRECTA? cional.
5) No hay ninguna alteracin somtica concomitante.
1) Aadir 1 mg de lorazepam dos veces al da.
2) Aadir 40 mg de propranolol una vez al da. 60. Dentro de las alternativas farmacolgicas para el tratamiento
3) Sustituir el haloperidol por 2 mg de risperidona. de deshabituacin a largo plazo del paciente alcohlico, NO
4) Disminuir la dosis de haloperidol a 10 mg/da. utilizamos:
5) Tranquilizar al paciente de lo transitorio de esta sintomato-
loga. 1) Naltrexona.
2) Diazepam.
56. Dentro de las opciones de abordaje de tipo psicosocial de la 3) Tiaprida.
esquizofrenia, se considera DESCARTADO en el momento actual: 4) Acamprosato.
5) Cianamida.
1) Terapia de modicacin de conducta.
2) Psicoeducacin. 61. De las relaciones siguientes entre frmacos y su uso en la depen-
3) Grupos de autoayuda de familias. dencia de opiceos, una es FALSA:
4) Psicoterapias dinmicas.
5) Centros de rehabilitacin psicosocial. 1) Naloxona tratamiento de la intoxicacin (sobredosis).
2) Metadona tratamiento de mantenimiento con opiceos.
3) Naltrexona tratamiento de deshabituacin con antagonistas.
TRASTORNOS 4) Clonidina tratamiento de deshabituacin sin opiceos.
RELACIONADOS CON SUSTANCIAS 5) Metadona - tratamiento de la abstinencia (desintoxicacin).

57. Dentro de las complicaciones asociadas al consumo agudo de 62. Un paciente aparece en su consulta demandando tratamiento
alcohol, hay una que est expresada de forma INCORRECTA: pues presenta dolores generalizados, sudoracin, hipertermia,
nuseas y vmitos; a la exploracin detectamos marcas cutneas
1) La intoxicacin idiosincrsica se produce tras un consumo en forma de ral y cicatrices de abscesos subcutneos en brazos
mnimo de alcohol. y piernas; no parecen existir antecedentes mdicos relevantes,
2) Existe buena correlacin entre la alcoholemia y los efectos qu actitud deberemos tomar?
neuropsiquitricos.
3) El tratamiento de la intoxicacin es sintomtico. 1) Remitir al paciente a la Unidad de Hospitalizacin Psiquitrica
4) La sobredosis de alcohol puede ser letal. ms cercana.
5) Con frecuencia vemos hiperglucemia como consecuencia 2) Derivar al paciente a un centro especializado en drogode-
de la intoxicacin. pendientes.
3) Administrarle midazolam intravenoso como medicacin
58. Un paciente ingresado por un traumatismo craneal presenta en sedante.
la noche del primer da de ingreso un episodio de agitacin psi- 4) Ponerle un goteo de naloxona tras 2 ampollas i.v. iniciales.
comotriz, encontrndose sudoroso y con temblor grosero; dice 5) Iniciar tratamiento con naltrexona y clonidina.
estar viendo cientos de insectos subiendo por las sbanas de la
cama y en la exploracin se encuentra desorientado en tiempo 63. Cul de los siguientes frmacos NO se usa para intentar
y espacio; uno de los hallazgos siguientes sera INCOMPATIBLE reducir el riesgo de recadas en consumidores crnicos de
con el cuadro que padece: cocana?

CTO Medicina C/Francisco Silvela, 106 28002 - Madrid Tfno. (0034) 91 782 43 30/33/34 E-mail: secretaria@ctomedicina.com www. ctomedicina.com 6
Test 1. vuelta Distancia
Psiquiatra
1) Bromocriptina. 1) Las alteraciones de las pruebas de neuroimagen sean alta-
2) Desipramina. mente inespeccas en ambos cuadros.
3) Propranolol. 2) Los pacientes depresivos tienden a mejorar por la noche,
4) L-dopa. mientras que los dementes suelen hacerlo por la maana.
5) Amantadina. 3) Los pacientes con demencia suelen exagerar sus fallos
cognitivos y los depresivos a minimizarlos.
4) Los cambios en las pruebas neuropsicolgicas son incon-
TRASTORNOS COGNOSCITIVOS: DELIRIUM, DEMENCIAS gruentes en la depresin y consistentes en las demencias.
Y AMNESIAS 5) La mitad de las llamadas pseuodemencias depresivas
desarrollarn, a medio o largo plazo, una verdadera
64. Un varn de 75 aos es ingresado para una reseccin prost- demencia.
tica transuretral; la primera noche tras la operacin se muestra
inquieto, creyendo reconocer en las enfermeras a los compaeros
que tuvo durante el Servicio Militar e intentando levantarse de OTROS TRASTORNOS MENTALES: PERSONALIDAD,
su cama; al ser retenido por la fuerza comienza a gritar llamando ALIMENTACIN, SUEO, INFANTILES
a la Legin pues dice estar secuestrado; qu NO ser lgico
encontrar acompaando a este sndrome? 69. Una chica de 17 aos es llevada al Servicio de Urgencias por sus
padres porque la han encontrado en su habitacin realizndose
1) Empeoramiento nocturno. cortes en los antebrazos. La paciente dice hacerse los cortes para
2) Alucinaciones visuales. sentir que existo y expresa sentimientos crnicos de vaco. Es
3) Agitacin psicomotriz. conocida en el Servicio de Urgencias por los numerosos intentos
4) Preservacin de la capacidad de atencin. autolticos que ha realizado mediante la ingestin de pastillas.
5) Fluctuaciones de la clnica. Reere tener atracones ocasionalmente y sus padres estn
asustados por su promiscuidad sexual. Durante la entrevista,
65. Qu antipsictico de los siguientes se recomienda para el dice en ocasiones que se quiere ir de vacaciones, para poco
control de la agitacin psicomotriz en los casos de delirium? despus ponerse a llorar y expresar deseos de suicidio. Cul
es el trastorno de personalidad que padece?
1) Tioridacina.
2) Sulpirida. 1) Personalidad obsesiva.
3) Zuclopentixol. 2) Personalidad antisocial.
4) Haloperidol. 3) Personalidad pasivo-agresiva.
5) Clorpromacina. 4) Personalidad lmite.
5) Personalidad depresiva.
66. En un hospital numerosos enfermos se encuentran en riesgo de
sufrir un delirium; en el estudio de este sndrome NO resulta til 70. Una persona que lleva una vida bastante marginal, con un reper-
una de las siguientes pruebas: torio limitado de intereses, una llamativa ausencia de relaciones
sociales y con una respuesta emocional escasa, presenta una
1) Hemograma. personalidad:
2) TC craneal.
3) Test de Rorscharch. 1) Esquizoide.
4) Anlisis de orina. 2) Esquizoafectiva.
5) ECG. 3) Esquizofrnica.
4) Esquizotpica.
67. Una paciente de 70 aos acude a su consulta reriendo frecuentes 5) Esquizofreniforme.
prdidas de objetos cotidianos y ocasional desorientacin en
calles poco familiares; qu prueba de entre las siguientes NO 71. Una adolescente se desmaya en clase de gimnasia, tras lo que
utilizara en el estudio inicial de este caso de deterioro cognitivo? es llevada al botiqun del instituto; all se detecta una tensin
arterial de 70/45 mmHg; la chica est muy delgada, detalle que
1) Hemograma y bioqumica plasmtica. no habamos notado hasta la exploracin fsica; confrontada con
2) Niveles de hormonas tiroideas. estos datos, cul de las siguientes conductas NO sera lgica
3) TC craneal. en esta paciente?
4) Polisomnografa (estudio de la latencia REM).
5) Mini examen cognoscitivo (MEC) de Lobo. 1) Preocupacin por el aspecto fsico y la esttica.
2) Alto rendimiento acadmico.
68. A su consulta acude un hombre de 65 aos acompaado por sus 3) Tendencia a vestirse con ropas ajustadas y ligeras.
familiares, que reeren haber observado en el paciente desde 4) Uso excesivo de laxantes.
hace un mes una prdida sustancial en la memoria reciente y a 5) Evitacin de las reuniones sociales en donde pueda verse
largo plazo. El propio enfermo se queja de la prdida de memoria forzada a comer en pblico.
y la resalta con gran angustia, dando la impresin de que exagera
las deciencias. Usted se plantea el diagnstico diferencial entre 72. Dentro de las caractersticas del atracn bulmico, NO se
demencia y pseudodemencia. Acerca del mismo, es FALSO que: encuentra:

CTO Medicina C/Francisco Silvela, 106 28002 - Madrid Tfno. (0034) 91 782 43 30/33/34 E-mail: secretaria@ctomedicina.com www. ctomedicina.com 7
Test 1. vuelta Distancia
Psiquiatra
1) Tensin creciente antes del atracn.
2) Relacin desproporcionada ingesta/tiempo.
3) Sensacin de prdida de control.
4) Seleccin de alimentos hipocalricos.
5) Aumento de la frecuencia por la noche.

73. Al revisar la polisomnografa de su paciente encuentra un


perodo con ondas muy lentas en el EEG, asocindose con un
tono muscular bajo y sin movimientos oculares; qu trastorno
del sueo se asocia a esta fase?

1) Bruxismo.
2) Apnea obstructiva del sueo.
3) Enuresis.
4) Sonambulismo.
5) Pesadillas.

74. La prdida brusca del tono muscular en respuesta a una emocin


es un sntoma casi patognomnico de un trastorno; cul es el
tratamiento del mismo?

1) Betabloqueantes.
2) Clonazepam.
3) Carbamacepina y valproico.
4) Clomipramina.
5) Sistemas de ventilacin positiva.

75. Dentro del autismo infantil, es frecuente observar alteraciones


conductuales, pero NO es tpico:

1) Estereotipias motoras.
2) Relativa preservacin del lenguaje.
3) Escaso contacto ocular.
4) Desinters por las relaciones con otros nios.
5) Intolerancia a los cambios de la rutina diaria.

76. En uno de los siguientes trastornos de la infancia, el tratamiento


con antipsicticos es de eleccin:

1) Dcit de atencin con hiperactividad.


2) Autismo.
3) Mutismo selectivo.
4) Sndrome de la Tourette.
5) Enuresis nocturna.

77. Carlos es un nio de 7 aos que sufre con frecuencia castigos


en clase por no aceptar la disciplina; pasa las horas de colegio
levantndose continuamente del pupitre y molestando a sus
compaeros, que han comenzado a hacerle el vaco pues dicen
que es muy bruto; en casa se comporta de forma parecida,
habiendo destrozado todos sus juguetes y sufriendo con fre-
cuencia accidentes, pues no mide correctamente el riesgo; qu
problema presenta con mayor probabilidad?

1) Retraso mental.
2) Trastorno disocial de la personalidad.
3) Conducta oposicionista.
4) Dcit de atencin con hiperactividad.
5) Depresin enmascarada.

CTO Medicina C/Francisco Silvela, 106 28002 - Madrid Tfno. (0034) 91 782 43 30/33/34 E-mail: secretaria@ctomedicina.com www. ctomedicina.com 8
Comentarios de Test a distancia 1. vuelta
Psiquiatra

Comentarios de Test a distancia 1. vuelta

Psiquiatra
TRASTORNOS NEURTICOS pulsivos), pero no vemos lo contrario (un paciente con crisis de angustia
no desarrolla fenmenos obsesivo-compulsivos).
Pregunta 1.-R: 2
Los trastornos primarios por ansiedad se pueden presentar de dos formas: Pregunta 2.-R: 1
Continua, como en el trastorno por ansiedad generalizada. En el trastorno por angustia (pnico) diferenciamos tres necesidades
Episdica, en forma de crisis; cuando las crisis tienen lugar siempre de tratamiento:
al enfrentarse a una determinada situacin decimos que el paciente El control de los sntomas agudos de una crisis de angustia (ataque
sufre una fobia; otros pacientes tienen crisis espontneas (llamadas de pnico) se realiza con benzodiacepinas por va oral o sublingual
crisis de angustia o ataques de pnico), sin relacin aparente con (aunque en Espaa NO se comercializa ninguna presentacin
estmulo alguno; cuando estas crisis espontneas se repiten y re- sublingual de benzodiacepinas). Es igualmente necesario ayudar
percuten en la vida de las personas, dando lugar a la aparicin de al paciente a frenar la hiperventilacin y explicarle la ausencia de
complicaciones psicolgicas, hablamos de trastorno por angustia consecuencias fsicas de la crisis y las caractersticas de su trastorno.
(o trastorno de pnico). La prevencin de nuevas crisis se realiza con antidepresivos, siendo
hoy en da de eleccin los ISRS (paroxetina o citalopram son los ms
No es raro que los trastornos por ansiedad se vayan confundiendo entre utilizados) por su mejor perl de efectos secundarios; sto no quiere
s cuando se cronican o complican; el ejemplo ms claro lo vemos en decir que no sean de utilidad los antidepresivos clsicos (tricclicos
las complicaciones que surgen cuando el trastorno por angustia se o IMAO). Tambin son ecaces las benzodiacepinas de alta potencia
prolonga en el tiempo. (alprazolam, clonazepam, lorazepam), pero el miedo a la posible depen-
dencia ha hecho que se trate de limitar su uso a las primeras semanas
del tratamiento, mientras el antidepresivo alcanza su mxima ecacia.
Finalmente, resulta imprescindible evaluar las distintas complicaciones
psicolgicas o psiquitricas del trastorno por angustia/pnico y ofrecer
un tratamiento especco si persisten una vez controladas las crisis.

La buspirona es un ansioltico NO benzodiacepnico (agonista parcial


serotoninrgico) con un perl ms adecuado para el trastorno por
ansiedad generalizada aunque su potencia ansioltica sea baja; carece
de efectos sedantes (no sirve para cortar una crisis de ansiedad), no
produce dependencia ni abstinencia y no interacciona con el alcohol;
tiene un inicio de accin lento (semanas) y no ha demostrado efectos
preventivos antipnico.

VENTAJAS DESVENTAJAS
No producen Tardan en hacer efecto (semanas)
Pregunta 1. Complicaciones del trastorno de pnico. dependencia Pueden producir un aumento
Antidepresivos
Tratan la posible inicial de la ansiedad (empezar a
De una forma algo articial, la clasicacin DSM tambin incluye dentro depresin asociada dosis bajas y subir lentamente)
de los trastornos por ansiedad los trastornos obsesivo-compulsivos (en Actan de forma
donde la ansiedad es secundaria a los fenmenos obsesivos y compul- rpida (das)
Benzodiace-
Escasos efectos Riesgo de abuso y dependencia
sivos) y los trastornos por estrs agudo y postraumtico (en donde la pinas
adversos (sedacin
ansiedad es uno de los muchos sntomas que aparecen ante una situacin transitoria)
estresante de intensidad mxima).
En el trastorno obsesivo-compulsivo, los pacientes pueden sufrir crisis Pregunta 2. Ventajas y desventajas de los tratamientos
de ansiedad (sobre todo cuando se les impide realizar los rituales com- preventivos para el pnico.

CTO Medicina C/Francisco Silvela, 106 28002 - Madrid Tfno. (0034) 91 782 43 30/33/34 E-mail: secretaria@ctomedicina.com www. ctomedicina.com 1
Comentarios de Test a distancia 1. vuelta
Psiquiatra
Pregunta 3.-R: 3 de impulsin (en las que el paciente tiene miedo de llevar a cabo esas
La mayora de los pacientes con agorafobia presentan adems crisis de ideas tan desagradables y evita situaciones potencialmente peligrosas
angustia espontneas (ataques de pnico); se considera, por tanto, que como el uso de cuchillos, el acercarse a las ventanas o el aproximarse
su agorafobia es secundaria. Pero tambin hay formas de agorafobia a los andenes del tren por el miedo a presentar un impulso frente al
primaria en las que no detectamos crisis espontneas. El tratamiento de que no se pueda resistir).
la agorafobia en s no es distinto: tcnicas psicolgicas de modicacin
de conducta (sobre todo la exposicin en vivo). La diferencia vendr dada Trastornos por tics (Tourette)
por la necesidad de aadir tratamiento farmacolgico para prevenir las Trastornos del control de los impulsos (juego patolgico/ludopata,
crisis de angustia en el primer caso. Las tcnicas psicolgicas basadas en cleptomana, piromana, tricotilomana, trastorno explosivo intermitente)
Trastornos de la conducta alimentaria
la modicacin de conducta son el tratamiento de eleccin en todas las
Algunos trastornos somatomorfos (hipocondra, dismorfofobia)
fobias. Suele realizarse primero un listado de todas las situaciones temidas Algunos trastornos de la personalidad (esquizotpico, lmite)
y una ordenacin en funcin del miedo asociado (jerarquizacin); poste- Algunas formas de toxicomana
riormente se programan ejercicios de exposicin en vivo, comenzando Algunas parafilias
en las situaciones ms fciles. Se han desarrollado otras tcnicas ms Algunos trastornos generalizados del desarrollo (Asperger, autismo atpico)
agresivas que buscan la exposicin brusca a la situacin fbica en su
mxima intensidad (inundacin o implosin), pero son peor toleradas. Pregunta 5. Presentacin clnica de los trastornos obsesivo-compulsivos.

Pregunta 4.-R: 3 Aunque en muchos casos los fenmenos compulsivos guardan una
El paciente con fobia social tiene miedo de aquellas situaciones en las relacin ms o menos lgica con las ideas obsesivas, en otros pacientes
que se expone a la opinin de otras personas. Lo que le preocupa no veremos rituales totalmente independientes, con una relacin ms bien
es, por tanto, el lugar en el que se encuentra, sino la reaccin de los mgica (p. ej., la necesidad de colocar las cosas en un determinado
dems ante su comportamiento. No es raro que estos pacientes limiten orden para que no suceda nada malo).
sus actividades sociales, pudiendo confundirse con otros pacientes que
tambin presentan retraccin social (agorafbicos que no salen de sus Pregunta 6.-R: 5
casas por el miedo a los desplazamientos, esquizofrnicos o depresivos El trastorno obsesivo-compulsivo idioptico suele aparecer al nal de
que restringen sus relaciones sociales por la prdida de inters asociada la adolescencia o principio de la juventud, de una forma insidiosa, y
a sus enfermedades, personalidades esquizoides sin inters real por la evoluciona con carcter crnico y de forma uctuante, agravndose en
conducta social). Quizs lo ms complicado sea diferenciarles de las perso- situaciones de estrs. Es raro que desaparezca por completo, pero tam-
nalidades evitativas o fbicas, con quienes comparten los mismos errores bin es excepcional que alcance una gravedad invalidante. La mayora
de pensamiento (preocuparse en exceso por la opinin de los dems), de los pacientes acaban por acostumbrarse a los fenmenos obsesivos
si bien se supone que la fobia social comienza en un momento dado y compulsivos, lo que puede explicar su escasa presencia en consulta
(en torno a los 20 aos) y las personas evitativas siempre han sido as. a pesar de su elevada prevalencia en poblacin general (cerca del 2%).
En las fobias sociales los frmacos pueden ayudar a los tratamientos Cuando debuta en la infancia se puede asociar con los trastornos por
psicolgicos de dos formas distintas; en las formas de fobia social res- tics (tanto los tics motores crnicos como el trastorno de la Tourette).
tringidas a situaciones poco frecuentes (p. ej., miedo a hablar en pblico Recientemente se ha descrito una asociacin entre algunas formas
delante de un auditorio) los bloqueantes de los receptores betaadrenr- infantiles de inicio agudo y fenmenos autoinmunes cercanos a la
gicos (propranolol, atenolol) disminuyen la respuesta ansiosa perifrica ebre reumtica (secundarios, por tanto, a infecciones estreptoccicas
y pueden permitir un mayor autocontrol de la ansiedad, aunque el [estreptococo betahemoltico o SGA] farngeas); se ha denominado
tratamiento de eleccin sera psicolgico, buscando mejorar la capa- PANDAS (siglas en ingls del trastorno neuropsiquitrico peditrico de
cidad de exponer en pblico; en la forma generalizada de fobia social origen autoinmune asociado al estreptococo) y se supone que comparte
se propone el tratamiento con antidepresivos como coadyuvantes de un mismo mecanismo patognico con la corea de Sydenham (lesin
la psicoterapia. En los ltimos aos los ISRS han desplazado a los IMAO mediada por autoanticuerpos frente a los ganglios basales).
en estos pacientes con fobia social grave. Cuando el trastorno obsesivo-compulsivo aparece en adultos o ancianos
es necesario buscar problemas neurolgicos como causa del mismo
Pregunta 5.-R: 4 (corea de Huntington, accidentes vasculares cerebrales, etc.).
Las dos principales formas de presentacin del trastorno obsesivo- En cualquier caso, tanto en las formas idiopticas como en las formas
compulsivo son: secundariasse implica a las mismas regiones cerebrales, loscircuitosde
La combinacin de ideas obsesivas relacionadas con la posible conexin entre corteza prefrontal, ganglios basales y tlamo. De hecho,
contaminacin o contagio de enfermedades y rituales compulsivos la posible utilidad de la psicociruga en algunos casos seleccionados se
de lavado excesivo o de evitacin del contacto con posibles fuentes basa en la interrupcin de alguna de las vas de estos circuitos.
contaminantes. El neurotransmisor ms relacionado con el trastorno obsesivo-com-
La combinacin de ideas obsesivas de duda o incertidumbre y rituales pulsivo es la serotonina, lo que va a ser utilizado en el tratamiento
compulsivos de comprobacin. farmacolgico.

Menos frecuentes son las obsesiones centradas en la necesidad de Pregunta 7.-R: 5


orden y simetra, que dan lugar a compulsiones de colocacin y Uno de los datos ms sorprendentes del trastorno obsesivo-compulsivo
ordenacin. Sin embargo, no es raro encontrar pacientes con otros es su respuesta selectiva a medicacin serotoninrgica; el tratamiento
fenmenos ms extraos, destacando las llamadas ideas de contraste farmacolgico de eleccin son los antidepresivos serotoninrgicos,
(ideas de contenidos agresivos o sexuales totalmente inapropiadas para pudiendo en ocasiones potenciarse su efecto con otras sustancias (litio,
la situacin en la que se encuentra el paciente y en desacuerdo con buspirona). En los casos refractarios y en aquellos en los que coexistan
sus deseos o creencias) que pueden conducir a la aparicin de fobias tics puede ser til aadir un antipsictico.

CTO Medicina C/Francisco Silvela, 106 28002 - Madrid Tfno. (0034) 91 782 43 30/33/34 E-mail: secretaria@ctomedicina.com www. ctomedicina.com 2
Comentarios de Test a distancia 1. vuelta
Psiquiatra
Dentro de los antidepresivos, el mejor estudiado es el antidepresivo Somatizadores: se quejan de numerosos sntomas (digestivos,
tricclico clorimipramina (o clomipramina), si bien sus efectos adversos neurolgicos, genitales, cardiorrespiratorios); tienden a cambiar
le han relegado a un segundo plano tras la aparicin de los ISRS. stos se de mdico cuando se les confronta con la posibilidad de un origen
han convertido en la medicacin de primera eleccin, teniendo en cuenta psicolgico de sus molestias; pueden abusar de la automedicacin
adems que en este trastorno suelen necesitarse dosis dos o tres veces (analgsicos) y corren el riesgo de someterse a pruebas diagnsticas
superiores a las habituales para la depresin y tiempos de tratamiento innecesarias y peligrosas; en su mayora son mujeres, que suelen
muy prolongados, lo que disminuye la efectividad del tratamiento con comenzar a tener sntomas en la juventud, siguiendo un curso
clorimipramina al aumentar los abandonos. Los IMAO se han usado crnico y uctuante; se utiliza a veces en estos casos el epnimo
para pacientes resistentes, mientras que las benzodiacepinas carecen de sndrome de Briquet. Una variante de somatizador ms limitada
de efecto antiobsesivo especco. en su expresin clnica es el paciente que se queja slo de dolor,
Junto con el tratamiento farmacolgico es necesario un tratamiento siendo ste desproporcionado para los hallazgos exploratorios.
psicolgico basado (como en las fobias) en las tcnicas de modicacin Hipocondracos y dismorfofbicos: estn preocupados por padecer
de conducta; en concreto, la tcnica preferida se denomina exposicin (a una enfermedad o sufrir un defecto fsico, respectivamente; aunque
la situacin obsesiva temida) con prevencin de respuesta (compulsiva). tambin pueden cambiar de mdico si no obtienen la atencin que
El psicoanlisis no obtiene resultados en los pacientes con este trastorno. creen necesaria, es ms raro en ellos la tendencia a la automedica-
En casos refractarios al tratamiento farmacolgico, la psicociruga (cin- cin o la realizacin de pruebas complementarias de riesgo; en el
gulotoma, capsulotoma bilateral anterior, tractotoma subcaudada) caso de los dismorfofbicos, el peligro fundamental son las posibles
puede ser ecaz. intervenciones estticas (mdicas o quirrgicas) que pueden dejar
secuelas; en ambos casos no veremos un predominio de ningn sexo
Pregunta 8.-R: 3 y el inicio suele ser ms tardo; los pacientes oscilan en la intensidad
Podemos agrupar los sntomas del trastorno por estrs postraumtico de sus preocupaciones, desde formas cercanas a los trastornos
en cuatro tipos: obsesivo-compulsivos hasta formas casi delirantes. Se propone para
Reexperimentacin del acontecimiento: pesadillas, imgenes tipo estos pacientes un tratamiento similar al de los trastornos obsesivo-
ash-back, recuerdos intrusivos. compulsivos (ISRS, clorimipramina), asociado a veces a antipsicticos.
Evitacin voluntaria (fbica) o involuntaria (amnesia psicgena) de Conversivos: se suelen presentar en Urgencias con signos neurol-
todo lo relacionado con el acontecimiento. gicos incongruentes a la exploracin fsica (convulsiones, parlisis,
Estado de hiperalerta: reacciones bruscas ante los ruidos, problemas anestesias, cegueras); casi siempre se identica un factor estresante
de concentracin, insomnio, irritabilidad. relacionado temporalmente con el inicio de los sntomas; tienden a la
Cambios emocionales: embotamiento, sensacin de corte vital. recuperacin espontnea, pero tambin a la recurrencia; en algunos
raros casos veremos formas de evolucin crnica; como en el caso
Cuando el trastorno por estrs no ha alcanzado un mes de duracin se de los somatizadores, suelen debutar en la juventud y hay un claro
denomina trastorno por estrs agudo, y suelen predominar los snto- predominio de las mujeres; en los pases desarrollados son cada vez
mas emocionales y disociativos. Se diagnostica un trastorno de estrs menos frecuentes. Parece que con el desarrollo cultural se produce
postraumtico cuando los sntomas superan el mes de duracin, con- una mayor tendencia a la somatizacin como expresin fsica del
siderndose que se cronica cuando alcanza los 3 meses de duracin. malestar psicolgico; ambas, conversin y somatizacin, junto con las
Se describen formas de inicio demorado que comienzan pasados varios variantes disociativas, componen las distintas formas de presentacin
meses del suceso, generalmente tras la exposicin a alguna situacin de la histeria, una enfermedad clsica dentro de la Psiquiatra que
que reactiva su recuerdo. parece traducir a sntomas fsicos o psiquitricos incongruentes la
Dentro de las complicaciones de estos trastornos destacan los sndromes respuesta de determinadas personas ante el malestar psicolgico.
depresivos y el abuso de sustancias.
De las posibles situaciones traumticas, aquellas que se deben a fen- Conviene NO confundir a estos pacientes (en los que los sntomas son
menos naturales (catstrofes como terremotos o inundaciones) pro- involuntarios y no existe enfermedad fsica real) con los pacientes PSI-
ducen menos problemas que las que se deben a la intervencin humana; COSOMTICOS (que presentan una enfermedad fsica real e involuntaria,
dentro de stas, los accidentes son menos traumticos que los delitos, en donde los factores psicolgicos inuyen en su origen o evolucin) ni
en los que una persona provoca voluntariamente un dao sobre otra; los con los pacientes que ngen VOLUNTARIAMENTE tener una enfermedad.
delitos sexuales y los secuestros producen estrs postraumtico en un Dentro de estos ltimos diferenciamos los simuladores, en los que existe
nmero muy elevado de casos (ms del 80% de las vctimas de violacin). una motivacin evidente de tipo econmico o legal, de los trastornos
No existe un tratamiento especco del estrs postraumtico; los anti- facticios, en los que la motivacin parece ser de tipo psicolgico, la ne-
depresivos (IMAO, ISRS) parecen obtener mejores resultados cuando cesidad de asumir la identidad de enfermo, incluso a costa de perjuicios
predominan los fenmenos de reexperimentacin y los cambios para su salud o su economa. La forma ms espectacular de trastorno
emocionales; las conductas de evitacin precisarn de un tratamiento facticio es el sndrome de Mnchausen, en el que los sntomas inventados
conductual adecuado; para reducir el nivel de alerta se pueden usar y los signos autoprovocados abarcan diferentes rganos y sistemas; la
benzodiacepinas (cuidado con el abuso de sustancias) y tcnicas de mayora de los facticios, sin embargo, son formas limitadas a sntomas
relajacin. Lo que s parece clara es la necesidad de detectar y tratar ms concretos, siendo ms difciles de detectar, pues los pacientes se
precozmente a estos pacientes, pues la cronicacin de los sntomas preocupan de no ser descubiertos y de parecerse lo ms posible a la
predice una mala respuesta al tratamiento. enfermedad real. Desde el punto de vista legal, la simulacin es un delito
evidente; los trastornos facticios pueden convertirse en un problema
Pregunta 9.-R: 3 legal cuando los pacientes provocan la enfermedad en terceras personas
Dentro de los pacientes que se presentan con sntomas fsicos para (hijos, esposo) para poder asumir la identidad de cuidador (trastornos
los que no se encuentra una explicacin mdica (SOMATOMORFOS) facticios por poderes). En la mayora de estos pacientes encontramos
tenemos: graves alteraciones de la personalidad, siendo refractarios al tratamiento.

CTO Medicina C/Francisco Silvela, 106 28002 - Madrid Tfno. (0034) 91 782 43 30/33/34 E-mail: secretaria@ctomedicina.com www. ctomedicina. 3
Comentarios de Test a distancia 1. vuelta
Psiquiatra
BZD tendrn mayor o menor potencia, pero a las dosis adecuadas
todas comparten los efectos anticonvulsivos, ansiolticos, hipnticos
y miorrelajantes. Sus principales diferencias son farmacocinticas
(velocidad de absorcin, vida media de eliminacin, presencia o no
de metabolizacin heptica).

Pregunta 12.-R: 2
Una de las ventajas de que las BZD tengan un lugar especco de accin ha
sido la posibilidad de disear un antagonista especco capaz de revertir
sus efectos en el caso de sobredosis; el umacenilo es el tratamiento
de eleccin en las intoxicaciones por BZD y se ha convertido, junto con
la naloxona (antagonista de los receptores opioides y tratamiento de
eleccin en las intoxicaciones por opiceos), en una medicacin bsica
de los servicios de urgencias. En el caso del resto de psicofrmacos
carecemos de antdotos especcos.
Las sobredosis de antidepresivos varan en su gravedad en funcin del
tipo de antidepresivo, siendo los ISR los frmacos ms seguros y los
Pregunta 9. Caractersticas de los fenmenos histricos. ADT los ms peligrosos (cardiotoxicidad), ocupando los IMAO un lugar
intermedio.
Pregunta 10.-R: 5 Las intoxicaciones con antipsicticos tienen un riesgo considerablemente
En los trastornos disociativos los pacientes presentan sntomas psicol- menor, dependiendo sus efectos en sobredosis de otras caractersticas
gicos (NO fsicos) extraos como cambios de comportamiento, amnesias (bloqueo histaminrgico, bloqueo alfaadrenrgico, efectos anticolinrgi-
o viajes sin sentido que no encajan en las enfermedades psiquitricas o cos) que varan enormemente de unos a otros; en el caso de presentarse
neurolgicas habituales y que responden de forma impredecible ante graves efectos extrapiramidales agudos, el tratamiento de eleccin son
los tratamientos habituales. los anticolinrgicos (biperideno, trihexifenidilo).
Como en el caso de los pacientes conversivos, es frecuente encontrar Dentro de los estabilizadores del humor, el litio es el ms peligroso en el
un factor estresante relacionado temporalmente con el sntoma y una caso de sobredosicacin, aunque la principal causa de intoxicacin por
tendencia a la desaparicin espontnea y a la recurrencia. Los sntomas litio no va a ser la toma voluntaria o accidental de un exceso de medicacin,
disociativos pueden aparecer en el contexto de otras enfermedades sino las interacciones con frmacos que alteran su eliminacin renal o la
psiquitricas, destacando su asociacin con los trastornos por estrs concurrencia de problemas hidroelectrolticos que tienen el mismo efecto;
(agudo y postraumtico) y con el trastorno de personalidad lmite. En el tratamiento de eleccin en caso de intoxicacin grave es la hemodilisis.
los ltimos aos se insiste en la frecuencia de antecedentes de abuso
sexual en la infancia de los pacientes con trastornos disociativos graves. Pregunta 13.-R: 5
Dependiendo de la cultura del pas encontramos diferentes formas de La mayora de las BZD se eliminan por va heptica mediante un doble
trastornos disociativos; as, las ms frecuentes en Espaa parecen ser los mecanismo: metabolizacin y conjugacin; el primero de estos proce-
episodios de amnesia, los ataques de nervios y el estupor psicgeno, sos se afecta gravemente en el caso de hepatopata, permaneciendo el
siendo excepcionales las fugas y anecdticos los casos de alteraciones segundo casi inalterado. Por eso las BZD, que slo se conjugan sin meta-
disociativas de la personalidad (personalidad mltiple); en pases me- bolizarse en el hgado, son las de eleccin en el caso de hepatopata (en
nos desarrollados veremos numerosos casos de trances y posesiones Espaa slo nos queda el lorazepam con una vida media de eliminacin
disociativas, y en EE.UU. llama la atencin la elevada frecuencia de casos corta, cercana a las 8 horas).
referidos con alteraciones disociativas de la personalidad.
La clasicacin DSM (no la CIE-10) incluye dentro de los trastornos diso- VIDA MEDIA
ciativos a los cuadros de despersonalizacin y desrealizacin crnicas;
Larga Corta
no hay que olvidar que estos sntomas de extraeza ante uno mismo o
el entorno pueden aparecer en numerosas enfermedades psiquitricas Dosis/da nica Mltiple
(trastornos por ansiedad, sobre todo en crisis, sndromes depresivos, Rebote de la ansiedad entre las dosis
No S
psicosis agudas, intoxicaciones por drogas) y en algunos trastornos neu- diurnas (con BZD de alta potencia)
ropsiquitricos, sobre todo los que afectan al lbulo temporal (epilepsia Rebote matinal de la ansiedad o el
No S
insomnio (con BZD de alta potencia)
parcial compleja, tumores, encefalitis); los casos en donde aparecen de
forma aislada y con evolucin crnica son excepcionales. Abstinencia No S
Restos diurnos en su uso como
S No
Pregunta 11.-R: 3 hipntico (resaca)
Las benzodiacepinas (BZD) interaccionan con unos lugares de accin Riesgo de acumulacin (en ancianos) S No
especcos (a veces llamados receptores benzodiacepnicos u omega),
dentro del complejo GABA-A; estos lugares de accin estn formados Pregunta 13. Ventajas e inconvenientes de las benzodiacepinas
por diversas subunidades proteicas, para las que existen numerosas en funcin de su vida media.
variantes, sin que se conozca todava la importancia funcional de cada
una de ellas. Desde el punto de vista farmacodinmico, todas las BZD Pregunta 14.-R: 1
hacen lo mismo: se unen a su lugar de accin y modican la anidad Existen otras BZD, de alta potencia, que tampoco se metabolizan de
del receptor gabargico de tipo A por el GABA, aumentando su acti- forma sustancial en el hgado (alprazolam, triazolam, midazolam) y cuya
vidad (modicacin alostrica positiva o potenciacin). Las diferentes vida media de eliminacin es quizs demasiado corta; en el caso de tria-

CTO Medicina C/Francisco Silvela, 106 28002 - Madrid Tfno. (0034) 91 782 43 30/33/34 E-mail: secretaria@ctomedicina.com www. ctomedicina.com 4
Comentarios de Test a distancia 1. vuelta
Psiquiatra
zolam y midazolam es de menos de 6 horas por lo que no se usan como Segn la gravedad de los sntomas, podemos ver EDM de intensidad leve,
ansiolticos; triazolam se utiliza para tratar el insomnio de conciliacin moderada o grave; las formas ms extremas de EDM llegan a presentar
(siempre en tandas cortas de pocas semanas para no producir tolerancia sntomas psicticos (delirios, sobre todo), por lo que se habla de EDM
y dependencia), y midazolam se usa como preanestsico y en procedi- con sntomas psicticos o depresin psictica (o depresin delirante).
mientos instrumentales potencialmente desagradables (cateterismos, Segn el patrn de sntomas tenemos EDM con sntomas melan-
colonoscopias) para relajar a los pacientes. clicos (o depresin endgena) y EDM con sntomas atpicos; sin
embargo, la mayora de los EDM no encajan en ninguno de estos dos
Como ansioliticos Betabloqueantes patrones, presentando sntomas inespeccos; parece que con las
Buspirona recadas sucesivas de la enfermedad los sntomas van adquiriendo
Antidepresivos un carcter ms endgeno y pueden alcanzar con mayor facilidad
Anticonvulsivos
Antipsicticos
una gravedad psictica; lo mismo parece ocurrir en los EDM que
debutan en la edad avanzada. Tambin podemos ver un EDM con
Como hipnticos Zolpidem, zoplicona, zalepln sntomas catatnicos (o depresin estuporosa).
Antihistamnicos
AD (y AP) sedantes Cuando el inicio del EDM guarda una relacin con el puerperio se
habla de EDM de inicio puerperal o depresin puerperal (o depresin
Como miorrelajantes Ciclobenzaprina posparto).
Como anticonvulsivos Fenobarbital, fenitona,
cido valproico, carbamacepina, La depresin neurtica (trmino clsico) se corresponde ms o menos
gabapentina, lamotriguina, topiramato
con la distimia de la clasicacin actual, no con los EDM.

Pregunta 14. Alternativas a las benzodiacepinas Pregunta 17.-R: 5


en sus diferentes indicaciones. Los sntomas depresivos atpicos incluyen tres caractersticas que, en
teora, predeciran una mala respuesta a medicacin antidepresiva:
La presencia de rasgos patolgicos de personalidad, en la lnea
TRASTORNOS AFECTIVOS. FRMACOS ANTIDEPRESIVOS dependiente o histrinica.
Y ESTABILIZADORES DEL HUMOR La existencia de un factor estresante asociado al inicio de los sntomas
(frecuentemente un problema sentimental).
Pregunta 15.-R: 4 La ausencia de sntomas melanclicos, observndose una inversin de
Clasicamos los sndromes afectivos en funcin de dos criterios: su los sntomas biolgicos (aparecen somnolencia excesiva y aumento
gravedad y su duracin. La gravedad se evala con diferentes escalas del apetito).
de sntomas (depresivos o manacos), aunque el principal criterio de
gravedad ser la repercusin que el sndrome clnico tenga sobre el Sin embargo, se describe una buena respuesta al tratamiento con IMAO,
funcionamiento del paciente. Atendiendo a la gravedad, diferenciamos siendo inferiores los resultados obtenidos con otros antidepresivos. Esta
los sndromes depresivos o manacos en MAYORES y MENORES. selectividad farmacolgica se ha relacionado con una mayor frecuencia
Para que un sndrome depresivo mayor sea considerado relevante se de alteraciones serotoninrgicas en estos pacientes. En general, los snto-
exige una duracin mnima de 2 semanas (EPISODIO DEPRESIVO MA- mas atpicos no implican una gravedad excesiva; de hecho, podemos ver
YOR), mientras que en el caso de la mana, la duracin se reduce a una sntomas atpicos tanto en los EDM como en las distimias. En cambio, los
semana o menos, si precisa hospitalizacin por su intensidad (EPISODIO sntomas melanclicos s se asocian a una mayor gravedad (y a un mayor
MANACO). En los sndromes depresivos menores se exige una mayor riesgo de suicidio, por tanto); en estos pacientes melanclicos se encuentran
duracin, debiendo persistir los sntomas depresivos 2 aos para aceptar ms alteraciones neurobiolgicas, por lo que su respuesta a los tratamien-
el diagnstico de DISTIMIA; en los sndromes manacos s se acepta la tos biolgicos es superior (tanto a antidepresivos como a electrochoque).
importancia de los cuadros menores de duracin recortada, diagnosti-
cndose de HIPOMANA a los sndromes manacos menores que duran Pregunta 18.-R: 5
al menos 4 das; la alternancia de sntomas hipomanacos y sntomas La depresin debuta en general en la edad adulta (sobre los 20 aos
depresivos menores durante un perodo de al menos 2 aos recibe el debutan los bipolares, en torno a los 40 aos los depresivos), siendo raros
diagnstico de CICLOTIMIA. los casos de inicio en la infancia o la vejez. Adems, en esas edades el
Los cuadros depresivos son mucho ms frecuentes que los manacos, diagnstico se complica al enmascararse la depresin bajo otros sntomas.
encontrndose en los primeros un claro predominio femenino, mien- En los ancianos no es raro que se oculte una depresin tras una serie
tras que en los segundos hay una cierta igualdad. De igual forma, los de quejas fsicas o de memoria, lo que en parte puede reejar la mayor
sndromes episdicos (de semanas o meses de duracin) son ms preocupacin por estos sntomas en este grupo de edad. Tambin es
frecuentes que los sndromes crnicos (de aos de duracin). De ah destacable la mayor frecuencia de sntomas psicomotores (inhibicin,
que el trastorno afectivo ms frecuente sea el episodio depresivo mayor. agitacin) y de sntomas psicticos; se han relacionado ambos sntomas
La melancola (o depresin endgena) es una variante de episodio con alteraciones en los ganglios basales, que podran deberse al deterioro
depresivo mayor que se identica por la presencia de unos sntomas cerebral asociado a la edad o a la presencia de factores de riesgo cere-
caractersticos; por tanto, aunque todos los casos de melancola son brovascular. La presencia de sntomas psicticos implica la necesidad de
episodios depresivos mayores, slo una pequea parte de los episodios modicar el tratamiento, aadiendo antipsicticos a los antidepresivos
depresivos mayores presentan sntomas melanclicos. o recurriendo al electrochoque. No hay que olvidar que la edad es por
s misma un factor que aumenta el riesgo de suicidio, lo que, aadido
Pregunta 16.-R: 4 a la presencia de un sndrome depresivo y a la gravedad psictica, que
Los episodios depresivos mayores (EDM) reciben diferentes adjetivos con frecuencia alcanza, explica el enorme riesgo de suicidio que tienen
calicativos en funcin de distintas caractersticas: las depresiones graves del anciano.

CTO Medicina C/Francisco Silvela, 106 28002 - Madrid Tfno. (0034) 91 782 43 30/33/34 E-mail: secretaria@ctomedicina.com www. ctomedicina. 5
Comentarios de Test a distancia 1. vuelta
Psiquiatra
En los nios, los sndromes depresivos van a ocultarse tras quejas fsicas (los Pregunta 21.-R: 2
nios pequeos carecen de lenguaje emocional y tienden a expresarse Uno de los problemas ms frecuentes ante el diagnstico de depresin
somticamente), trastornos de conducta o descenso del rendimiento es la tendencia casi automtica a buscar un factor psicosocial estresante
acadmico; en los adolescentes, los trastornos de conducta, incluido que consideramos desencadenante. Para empezar, no hay que olvidar
el abuso de sustancias, pueden enmascarar una depresin. No hay que que muchas personas se enfrentan a diario a situaciones estresantes muy
olvidar que, en nios y adolescentes el estado de nimo predominante similares, pero slo una parte de ellas desarrollan un sndrome depresivo;
durante una depresin va a ser con frecuencia irritable ms que triste, hay una serie de factores individuales que aumentan el riesgo de presen-
lo que dicultar el diagnstico. tar una depresin, entre los que destaca la presencia de antecedentes
familiares, los antecedentes personales de depresin y determinados
Pregunta 19.-R: 3 rasgos de personalidad. Los factores estresantes parece que actan
Dentro de los sndromes depresivos, los sntomas fsicos son los ms fciles de una forma inespecca, aumentando la probabilidad de presentar
de diagnosticar, pues tienen un carcter objetivo; sin embargo, carecen de un sndrome psiquitrico en funcin de la predisposicin individual.
especicidad diagnstica al presentarse tambin en casi todas las enferme- Adems, no existe una buena correlacin entre la presencia o ausencia
dades psiquitricas y mdicas; por sto, en los pacientes con enfermedades de un factor estresante y la duracin, la intensidad o las caractersticas
psiquitricas o mdicas coexistentes, habr que recurrir a sntomas ms del sndrome depresivo asociado; la mitad de los pacientes con EDM
subjetivos (estado de nimo, pensamientos) para asegurar el diagnstico. no reeren acontecimiento estresante alguno en las semanas previas
La alteracin del sueo ms frecuente en una depresin es el insomnio de al inicio de su depresin, y su evolucin no es diferente de la evolucin
caractersticas inespeccas (problemas de conciliacin con despertares de los pacientes que s reeren un acontecimiento causal.
nocturnos); cuando aparece tendencia al despertar precoz sugiere una Hoy en da se reserva el trmino trastorno adaptativo con sntomas
depresin melanclica (EDM con sntomas melanclicos o depresin depresivos para aquellos pacientes que presentan sntomas depresivos
endgena), sobre todo si se asocia con una variacin diurna del estado de de intensidad leve en respuesta a una situacin estresante que produce
nimo en la forma de empeoramiento matutino y mejora vespertina. Los reacciones similares en otras personas; estos sntomas se resuelven
pacientes con sntomas atpicos suelen referir una desagradable necesidad espontneamente en muchos casos y no suelen precisar tratamiento
de dormir durante el da, quejndose adems de un cansancio extremo. farmacolgico, salvo que su intensidad aumente.
En lo que hace referencia al apetito y al peso, lo ms frecuente es ver una El trmino depresin reactiva engloba, por tanto, desde episodios
disminucin de ambos; en los pacientes atpicos habr aumento del ape- depresivos mayores, a distimias y trastornos adaptativos.
tito y del peso, describindose una curiosa tendencia a comer mayores
cantidades de hidratos de carbono (dulces) de lo habitual para el paciente. Pregunta 22.-R: 3

Pregunta 20.-R: 4
En muchos pacientes depresivos el estado de nimo ucta en funcin de
diversos factores. Solamente en las depresiones ms graves el estado de
nimo permanece inalterable todo el da, con una ausencia de reactividad
ante lo que le rodea (anestesia emocional). En los sndromes depresivos
leves los pacientes conservan cierta capacidad de mejorar cuando las
circunstancias externas les son favorables (se dice que preservan la
reactividad); as tienden a notarse mejor cuanto ms activos estn o
cuando estn acompaados; como en general estamos ms ocupados
por las maanas, puede dar la impresin de que estos pacientes siguen
un ritmo circadiano, lo que biolgicamente no es cierto.
En los cuadros melanclicos el paciente puede encontrarse algo mejor
por la noche (se conoce como mejora vespertina), pero lo que s va a
referir con frecuencia es que est mucho peor por la maana, nada ms
despertarse (empeoramiento matutino); la asociacin entre despertar
precoz y empeoramiento matutino es altamente especca de melancola
(o depresin endgena), y se ha relacionado con alteraciones en los Pregunta 22. Relacin entre la intensidad de la depresin
marcapasos endgenos pues sigue un ritmo circadiano muy parecido al y la modalidad de tratamiento.
ritmo del cortisol (que con frecuencia est alterado en estos pacientes).
La respuesta selectiva a los IMAO frente a tricclicos es una de las carac- En los sndromes afectivos ms graves los pacientes pueden presentar
tersticas de las depresiones atpicas. sntomas psicticos; en general estos sntomas son ms probables en
La asociacin entre depresin y estaciones no es tan frecuente como se los cuadros manacos que en los depresivos; los delirios son mucho
cree popularmente; tan slo un 15% de los pacientes con enfermedades ms frecuentes que las alucinaciones, por lo que usamos los trminos
afectivas recurrentes (depresivas o bipolares) presentan un ritmo estacio- depresin (o mana) psictica o delirante de forma similar.
nal claro, que se repite de ao en ao. Lo habitual en estos casos es que En la mayora de los pacientes con estos sntomas encontramos una
los EDM aparezcan en otoo y los episodios manacos en verano. En los relacin lgica entre el contenido de los delirios y alucinaciones y su
pases nrdicos se ha descrito una variacin del ritmo estacional en los estado de nimo (sntomas psicticos congruentes, ideas deliroides o
pacientes bipolares con EDM en invierno (frecuentemente con sntomas ideas delirantes secundarias); si estn deprimidos, su visin pesimista
atpicos) y episodios manacos en verano. Este cambio se ha puesto en se extiende hacia el pasado (ideas de culpa), el presente (ideas de mi-
relacin con la duracin del fotoperodo (muy corta en invierno, muy nusvala y enfermedad) y el futuro (ideas de ruina); si estn manacos,
larga en verano), proponindose el tratamiento con fototerapia con su optimismo y autoconanza puede llegar a los delirios de grandeza.
carcter preventivo o curativo. En algunos casos, los delirios y alucinaciones se alejan de los contenidos

CTO Medicina C/Francisco Silvela, 106 28002 - Madrid Tfno. (0034) 91 782 43 30/33/34 E-mail: secretaria@ctomedicina.com www. ctomedicina.com 6
Comentarios de Test a distancia 1. vuelta
Psiquiatra
lgicos (delirios de perjuicio, delirios de control), planteando el diagns- Otras diferencias entre la enfermedad bipolar y la depresiva son de tipo
tico diferencial con la esquizofrenia y los trastornos esquizoafectivos; no epidemiolgico (predominio de mujeres en la enfermedad depresiva,
hay un acuerdo acerca de la importancia de estos sntomas psicticos inicio ms temprano en la enfermedad bipolar, mayor riesgo [relativo]
incongruentes con el estado de nimo, variando la forma de clasicacin de suicidio en la enfermedad bipolar). Se han realizado numerosas
de estos pacientes desde su inclusin en el trastorno esquizoafectivo comparaciones entre las recadas depresivas de ambas enfermedades;
(CIE) hasta el mantenimiento del diagnstico de trastorno afectivo (DSM). parece que las depresiones bipolares tienden a ser algo ms cortas que las
unipolares (6 meses frente a 12 meses), aunque con una mayor frecuencia
Pregunta 23.-R: 3 de recidivas a lo largo de la vida (6-9 frente a 2-3). En general, los sntomas
La respuesta habitual de un paciente deprimido al tratamiento con son muy parecidos, aunque se describen ms sntomas atpicos en las
antidepresivos es la vuelta a la normalidad. De hecho, si una persona formas bipolares (sobre todo en las bipolares-2) y una mayor tendencia
que no est deprimida toma estos medicamentos NO va a notar un au- a la agitacin psicomotora en las depresiones unipolares. No hay que
mento del estado de nimo, sino simplemente efectos secundarios (los olvidar que en la enfermedad bipolar podemos ver cualquier variante
antidepresivos NO son euforizantes y los euforizantes, como la cocana, de EDM (melanclico, atpico, psictico, catatnico) y diferentes formas
NO tienen efecto antidepresivo). de episodios manacos (mana clsica eufrica, mana atpica disfrica,
En algunos casos los pacientes sufren un cambio radical de su estado hipomana, episodios mixtos).
de nimo en respuesta al tratamiento, alcanzando una intensidad hipo-
manaca o manaca a gran velocidad. Esta respuesta tan rpida es tpica Pregunta 25.-R: 3
de los pacientes bipolares, recomendndose en ellos la retirada del fr- La mayora de los pacientes bipolares debutan con un EDM, siendo
maco antidepresivo en cuanto se consigue la normalizacin para evitar de especial inters la posibilidad de predecir una evolucin bipo-
el cambio de fase (a diferencia de los pacientes depresivos, en los que lar desde el primer episodio. El principal factor de riesgo para ser
se mantiene el mismo frmaco 6 meses a la misma dosis). Sin embargo, bipolar es tener antecedentes familiares de esa enfermedad, dado
vemos pacientes que tienen respuestas maniformes al tratamiento con que la enfermedad bipolar tiene la heredabilidad ms alta de toda
antidepresivos sin que hayan presentado nunca episodios manacos la psiquiatra (cercana al 90%). En las mujeres, un EDM puerperal
espontneos; ocialmente no se pueden catalogar como bipolares, evolucionar hacia una enfermedad bipolar en cerca del 80% de los
pero debera tenerse en cuenta en ellos la posibilidad de tratamiento casos. El debut en la adolescencia tiende a ser ms frecuente en las
de prevencin de recadas con litio. formas bipolares (pero NO es determinante). La respuesta anormal-
mente rpida al tratamiento antidepresivo y la aparicin de cuadros
Pregunta 24.-R: 5 maniformes secundarios al mismo, son tambin dos caractersticas
A mediados del siglo XX se pudo diferenciar la enfermedad bipolar de la que sugieren bipolaridad.
enfermedad depresiva atendiendo a la mayor frecuencia de antecedentes Cuando los pacientes debutan con un episodio manaco, el diagnstico
familiares afectivos en la primera; de hecho, hay autores que ante un es inmediato; los varones tienen una mayor tendencia a presentar este
paciente depresivo con antecedentes familiares de enfermedad bipolar tipo de comienzo y a tener ms fases manacas, lo que con frecuencia
proponen ya el tratamiento prolctico con litio, entendiendo que ser plantea problemas de diagnstico con la esquizofrenia (varones, jve-
cuestin de tiempo el que presente un episodio maniforme. nes, conducta desorganizada y agitada, delirios y alucinaciones) y los
comportamientos antisociales (varones, jvenes, conductas arriesgadas,
abuso de sustancias).
FORMAS UNIPOLARES FORMAS BIPOLARES
Pregunta 26.-R: 3
Prevalencia-vida Alta (15%) Baja (1%)
Actualmente se aceptan dos variantes principales de trastorno afectivo
Distribucin sexual Mujeres > hombres Mujeres = hombres bipolar (TAB). En el TAB-1, los pacientes tienen episodios manacos o
Edad de inicio Tarda (> 40 aos) Joven (< 30 aos) mixtos, asociados casi siempre a episodios depresivos; como se ve,
Clase social Baja? Alta? cabe la posibilidad de que un paciente tenga slo episodios manacos
o mixtos, pero es algo excepcional y no parece que evolucionen de
Sana Sana
Personalidad previa forma diferente a los pacientes manaco-depresivos.
(rasgos melanclicos) (rasgos ciclotmicos)
En el TAB-2 veremos EDM junto a episodios hipomanacos; estas
Antecedentes Muy frecuentes
familiares
Frecuentes (unipolares)
(bipolares, unipolares)
hipomanas son difciles de diagnosticar y no es raro que estos
pacientes se clasifiquen errneamente como depresivos, privn-
Recadas Pocas (1-3) Muchas (6-9)
doles del tratamiento con estabilizadores, lo que explica su peor
Duracin del episo- pronstico a largo plazo (mayores tasas de suicidio, ms tendencia
Larga (12-24 meses) Corta (6-9 meses)
dio depresivo a la ciclacin rpida); en los pacientes con TAB-2 vemos un ligero
Alteraciones predominio femenino, mientras que en los TAB-1 hay un equilibrio
psicomotoras Agitacin Inhibicin entre sexos.
en la depresin
Se acepta slo una forma menor de trastorno bipolar, la ciclotimia, de-
Riesgo de suicidio Menor Mayor nida por la alternancia de sntomas hipomanacos y sntomas depresivos
Induccin de ma- leves durante un perodo mnimo de 2 aos.
No S
na/hipomana La alternancia de episodios depresivos invernales y episodios maniformes
Prevencin de en verano caracteriza a la forma estacional (nrdica) del TAB que suele
Antidepresivos Estabilizadores
recadas tratarse de un TAB-2 (EDM ms hipomana).
La aparicin de un EDM sobre una distimia previa se conoce a veces como
Pregunta 24. Comparacin entre la depresin recurrente depresin doble. Implica una mayor gravedad que los EDM aislados, con
y el trastorno bipolar. menor tendencia a la recuperacin.

CTO Medicina C/Francisco Silvela, 106 28002 - Madrid Tfno. (0034) 91 782 43 30/33/34 E-mail: secretaria@ctomedicina.com www. ctomedicina. 7
Comentarios de Test a distancia 1. vuelta
Psiquiatra
y de intento de suicidio para referirse a los pacientes que se autolesionan
por otras razones.

Pregunta 28.-R: 2
Se llama depresin secundaria o exgena a todo sndrome depresivo
para el que se encuentra una causa mdica, txica o farmacolgica.

En la prctica destacaremos tres causas especialmente frecuentes:


Frmacos y drogas: aquellos frmacos que alteran la funcin noradre-
nrgica central (betabloqueantes, reserpina, alfa-metil-dopa, clonidi-
na), frmacos con actividad esteroidea (estrgenos, progestgenos,
corticoides, ACTH), intoxicacin crnica por drogas sedantes (alcohol,
opiceos, benzodiacepinas) y abstinencia de drogas estimulantes
(cocana, anfetaminas).
Enfermedades endocrinolgicas: sobre todo el hipotiroidismo, pero
tambin el hipertiroidismo (sobre todo en ancianos), el hiperparati-
roidismo o los trastornos adrenales (Cushing, Addison).
Enfermedades neurolgicas: tanto las enfermedades degenerativas
que afectan de forma ms o menos difusa a todo el sistema nervioso
central (Alzheimer, Parkinson) como algunas enfermedades localiza-
Pregunta 26. Espectro bipolar. das (tumores, accidentes vasculares, abscesos) que lesionan sobre
todo las regiones frontales.
Pregunta 27.-R: 4
En psiquiatra diferenciamos dos conceptos referidos al suicidio que La isoniacida tiene actividad IMAO y fue uno de los primeros anti-
popularmente se mezclan de forma inadecuada. depresivos usados en psiquiatra a principios de los aos 50, siendo
En primer lugar hablamos de SUICIDIO cuando una persona se autole- posteriormente sustituido por otros frmacos con menor toxicidad; por
siona de forma voluntaria y deliberada con la intencin de acabar con tanto, comparte con ellos la capacidad de producir cuadros maniformes.
su vida. Por otra parte, hablamos de AUTOLESIN (sin ms) cuando una
persona se provoca algn tipo de dao fsico sin que existiera un deseo Pregunta 29.-R: 5
explcito de muerte. Estas autolesiones son mucho ms frecuentes que La utilidad de los antidepresivos va mucho ms all de los sndromes
los verdaderos intentos de suicidio (100 veces o ms) y, en general, se depresivos, de ah que en la actualidad se preera denominarles por
pueden dividir en aqullas que tienen una nalidad MANIPULADORA su mecanismo de accin (ISRS, IMAO, inhibidores no selectivos de la
(distraer o llamar la atencin del entorno, conseguir algn tipo de recaptacin).
benecio) y aqullas que tienen un carcter ms IMPULSIVO (como las Dentro de los trastornos psiquitricos van a usarse antidepresivos en casi
reacciones en cortocircuito de los pacientes con personalidad lmite). todos los trastornos por ansiedad (en el pnico para prevenir las crisis, en
Ante cualquier autolesin no accidental debe solicitarse una valoracin la fobia social grave, en el trastorno obsesivo-compulsivo, en el trastorno
psiquitrica que determinar la intencionalidad de la misma. La mayora por estrs postraumtico y en el trastorno por ansiedad generalizada).
de las autolesiones no suicidas se presentan en jvenes, sobre todo mu- Tambin los utilizamos para frenar la impulsividad en pacientes con tras-
jeres, que se enfrentan a una situacin estresante ms o menos grave, tornos de la conducta alimentaria (atracones bulmicos), trastornos del
careciendo de otros recursos psicolgicos para abordarla (con frecuencia control de los impulsos (juego patolgico o ludopata), personalidades
hay rasgos patolgicos de personalidad); suelen elegirse mtodos de baja tipo lmite o trastornos por abuso de sustancias (alcoholismo). Dentro
letalidad (intoxicacin con medicamentos, cortes con armas blancas) y de estos ltimos pacientes, los antidepresivos van a tener cierta capa-
no se prepara el acto con antelacin, siendo relativamente fcil el rescate cidad de aliviar el sndrome de abstinencia de la cocana y parece que
(a veces se llevan a cabo en presencia de la propia familia); tras su aten- protegen de la neurotoxicidad inducida por xtasis y otras anfetaminas
cin mdica en Urgencias y la valoracin psiquitrica se les remite a los de diseo. En algunos trastornos psiquitricos propios de la infancia
dispositivos ambulatorios, siendo excepcional la necesidad de ingreso. (enuresis, trastorno por dcit de atencin) pueden utilizarse, aunque
En cambio, la mayora de los intentos de suicidio se dan en perso- no sean el tratamiento de primera eleccin. Por la capacidad sedante
nas mayores de 50 aos (una excepcin notable son los intentos de de algunos antidepresivos se proponen stos como alternativa a las
suicidio que presentan los jvenes con esquizofrenia), varones que benzodiacepinas en algunos casos de insomnio.
sufren enfermedades psiquitricas bien denidas (depresin, abuso de Ya en un terreno ms neuropsiquitrico, vemos cmo los antidepresivos
sustancias, esquizofrenia), asociadas o no a otros factores adversos de tricclicos se utilizan para la prevencin de la cefalea tensional y el trata-
tipo mdico (enfermedades crnicas, dolorosas, incapacitantes) o social miento de dolores neuropticos crnicos. Tienen adems la capacidad
(viven solos, han perdido una relacin signicativa recientemente, pasan de reducir la frecuencia de episodios de cataplejia en la narcolepsia. El
por dicultades econmicas); suelen utilizarse mtodos ms agresivos efecto antihistamnico de la doxepina supera a la mayora de los anti-
(precipitacin desde gran altura, ahorcamiento, armas de fuego) y histamnicos ociales, y no es raro que se use para el tratamiento del
se planica mejor el acto, evitando ser descubiertos; si sobreviven al prurito idioptico.
intento de suicidio, suelen ingresar y es necesario adoptar una serie
de precauciones para que no vuelvan a intentar suicidarse durante el Pregunta 30.-R: 3
ingreso (sedacin, contencin mecnica, vigilancia constante). A la hora de enfrentarnos al tratamiento de un sndrome depresivo,
Sin embargo, conviene tener en cuenta que algunos libros hablan de vamos a decidirnos por un tipo de tratamiento u otro en funcin de la
suicidio consumado para referirse a los pacientes autnticamente suicidas intensidad de los sntomas. Cuando un sndrome depresivo se acerca a

CTO Medicina C/Francisco Silvela, 106 28002 - Madrid Tfno. (0034) 91 782 43 30/33/34 E-mail: secretaria@ctomedicina.com www. ctomedicina.com 8
Comentarios de Test a distancia 1. vuelta
Psiquiatra
la intensidad de EDM, la probabilidad de respuesta a frmacos antide- Con los modernos ISRS todo este proceso de tratamiento se simplica,
presivos (AD) supera el 60%, siendo mayor an en los casos de EDM con pues la dosis inicial suele ser la dosis suciente (no hay que subir poco
sntomas melanclicos. En las formas extremas de depresin (depresin a poco, para que el paciente se acostumbre a los efectos adversos como
psictica) ser necesario combinar AD con antipsicticos o recurrir al con los AD tricclicos), tanto para el tratamiento agudo y de continuacin,
tratamiento con electrochoque. En las formas menores de depresin la como para el tratamiento de mantenimiento a largo plazo.
respuesta a frmacos ronda el 40% y se acerca mucho a la alcanzada con Se discute si alguno de los AD ms modernos (venlafaxina o mirtaza-
placebo, logrndose como mucho un alivio sintomtico. pina) tendra un efecto algo ms rpido aunque las diferencias no son
Una vez decidida la necesidad de poner un AD, la eleccin entre los clnicamente relevantes.
distintos compuestos depender de:
Los antecedentes de respuesta a frmacos en episodios previos. Pregunta 32.-R: 5
La presencia de otras enfermedades que contraindiquen el uso de Los AD tricclicos (ADT) son los AD ms antiguos y, por tanto, aqullos
un AD en concreto (por ejemplo, una cardiopata contraindicara el de los que se dispone de ms experiencia. De hecho, la imipramina
uso de AD tricclicos). se considera el AD patrn con el que deben compararse los dems
La toma de alguna medicacin que interaccione con el AD. antidepresivos. Los ADT logran su efecto antidepresivo aumentando la
El perl de efectos adversos del AD que puedan provocar su rechazo disponibilidad de las principales monoaminas (noradrenalina, serotonina
por el paciente. y dopamina) al inhibir su recaptacin.
Los antecedentes de respuesta a un AD en concreto en un familiar Sin embargo, no lo hacen de una forma selectiva, sino que tambin
de primer grado. bloquean numerosos receptores, cada uno de los cuales va a producir
El patrn de sntomas depresivos (por ejemplo, los sntomas atpicos una serie de efectos secundarios. No todos los ADT son iguales en esta
obligan a usar IMAO). caracterstica; los hay con potentes efectos anticolinrgicos (amitriptilina,
La comodidad de su uso. clorimipramina) y los hay con menos problemas a ese nivel (nortriptilina,
El precio del frmaco. desipramina); estos efectos muscarnicos son muy mal tolerados, espe-
cialmente por pacientes ancianos. Asimismo, tenemos ADT muy sedantes
Teniendo en cuenta que la mayora de los pacientes se presentan sin por su efecto antihistamnico (amitriptilina, doxepina, clorimipramina)
haber tomado nunca medicacin AD o no la recuerdan, los ISRS renen y otros ms activadores con escasa capacidad sedante (imipramina).
las caractersticas idneas para convertirse en los AD de primera eleccin. Lo que suele ser necesario en todos los ADT es comenzar con dosis
muy bajas e ir subiendo poco a poco, para lograr cierta tolerancia a los
Pregunta 31.-R: 3 efectos adversos. Un problema adicional de los ADT son sus efectos
Los antidepresivos no mejoran la depresin rpidamente, si no que tienen cardiovasculares; por su capacidad de bloqueo adrenrgico pueden
un efecto a medio plazo, tardando varias semanas (4-6) en conseguir la causar hipotensin ortosttica y por su accin parecida a la quinidina
remisin de los sntomas, aunque previamente ya se hayan detectado sobre la conduccin cardaca cabe la posibilidad de que produzcan
signos de respuesta. arritmias potencialmente letales. La intoxicacin con ADT supone un
Una vez alcanzada la remisin, se propone la necesidad de continuar serio problema en los pacientes depresivos con ideas suicidas.
con el mismo tratamiento (el mismo AD, a la misma dosis) un mnimo Sin embargo, todava se utilizarn atendiendo a tres ventajas:
de 6 meses para prevenir la recada del episodio. Su precio (7-10 veces inferior al precio de los nuevos AD), lo que
Pasado ese tiempo (que dependiendo de diversos factores, como la resulta signicativo en pases con menor nivel de desarrollo.
mayor edad, la gravedad del episodio o el nmero de episodios previos, La posibilidad de controlar el cumplimiento mediante los niveles
podr alargarse), se considera que el paciente se ha recuperado del plasmticos (sobre todo con imipramina).
episodio depresivo. Entonces se podr retirar el AD (casi siempre de Su posible mayor potencia antidepresiva (que se explicara por el
forma progresiva, en unas semanas para minimizar posibles sntomas hecho de que la mayora de los ADT actan a un doble nivel: nora-
abstinenciales), salvo a los pacientes con alto riesgo de recurrencia drenrgico y serotoninrgico).
(como aqullos que hayan sufrido numerosos episodios), a los que se
les propondr tratamiento indenido de mantenimiento (que suele En realidad, salvo la imipramina en depresiones resistentes a otros AD,
realizarse en la dosis mnima posible para reducir los efectos adversos la amitriptilina en el control del dolor crnico y la clorimipramina en el
y mejorar as el cumplimiento). TOC, el resto de ADT se usan de forma anecdctica.

Pregunta 32. Esquema de los efectos de los antidepresivos tricclicos


Pregunta 31. Fases del tratamiento de la depresin. y posibles efectos secundarios.

CTO Medicina C/Francisco Silvela, 106 28002 - Madrid Tfno. (0034) 91 782 43 30/33/34 E-mail: secretaria@ctomedicina.com www. ctomedicina. 9
Comentarios de Test a distancia 1. vuelta
Psiquiatra
Pregunta 33.-R: 1 las conservas y los salazones) lentamente se han ido abandonando; en la
Al principio de la dcada de los aos 80, los ISRS irrumpen en el mercado actualidad slo nos queda un IMAO clsico en Espaa (tranilcipromina),
psicofarmacolgico con tres ventajas evidentes: aunque en otros pases (Francia, Inglaterra, EE.UU.) an podemos encon-
Un perl de efectos secundarios mucho ms tolerable. trar otros IMAO (la fenelcina es el ms usado en el mundo).
Una mayor comodidad de uso, al no tener que aumentar lentamente Estrictamente hablando, habra slo una indicacin para el uso de
la dosis, ni fraccionarla en varias tomas diarias. IMAO como primera opcin: la depresin con sntomas atpicos.
La casi ausencia de letalidad en el caso de sobredosis. Sin embargo, incluso en estos casos es cada vez ms frecuente el
uso de ISRS.
En apenas 10 aos desplazan casi por completo a los ADT y a los IMAO Las alternativas a los IMAO clsicos son los IMAO reversibles de la MAO-
en el terreno de la depresin, y poco a poco van conquistando otras A (como la moclobemida) que lamentablemente tienen una potencia
indicaciones (pnico, trastorno obsesivo, bulimia, fobia social) hasta antidepresiva muy baja y los IMAO selectivos de la MAO-B (como la
alcanzar actualmente el 90% de las prescripciones de antidepresivos en selegilina o la rasagilina) que se usan para la enfermedad de Parkinson
Espaa. Se consideran frmacos de primera eleccin en el tratamiento de pues prcticamente slo consiguen elevar la dopamina.
la depresin (depresin mayor, distimia) y de los trastornos de ansiedad.
Sin embargo, tienen tres aspectos discutibles: Pregunta 35.-R: 3
Su precio es superior al de los antidepresivos tricclicos, aunque los La terapia electroconvulsiva (TEC) fue la primera estrategia realmente
estudios de coste-efectividad han demostrado que la diferencia de ecaz para el tratamiento de la depresin; se comenz a utilizar a nales
precio del frmaco se compensa por otros costes relacionados con de los aos 30, y hasta la revolucin farmacolgica de los aos 50-60
la depresin (das de baja laboral, necesidad de ingreso, necesidad fue el instrumento preferido para el tratamiento agudo de la depresin.
de medicacin adicional para tratar efectos adversos). Adems, la Tras unos aos de rechazo, ms por motivos ideolgicos que por razo-
aparicin de innumerables genricos ha bajado mucho su precio. nes cientcas, desde el nal de la dcada de los aos 80 asistimos a un
Se discute si tienen una menor potencia antidepresiva (debida quizs a renacimiento de la TEC. La TEC tiene tres ventajas sobre los frmacos AD:
su efecto exclusivamente serotoninrgico), aunque los estudios sobre Es ms potente en su accin antidepresiva, lo que explica su principal
efectividad en condiciones reales (no de ensayo clnico) demuestran indicacin, la depresin mayor resistente a AD, y el que, en ocasiones,
que los pacientes tienden a abandonar el tratamiento con ADT o no se plantee su uso como primera opcin en la forma ms grave de
alcanzan dosis adecuadas por culpa de los efectos adversos, con lo depresin, la depresin psictica o delirante.
que la terica mayor ecacia de los ADT se pierde, igualndose su Logra su efecto antidepresivo ms rpido que los frmacos (en 2-3
efectividad a la de los ISRS. semanas cuando se aplica a das alternos), lo que explica su utilidad
Van a presentar numerosas interacciones farmacocinticas en los en las depresiones estuporosas o catatnicas (con graves problemas
citocromos P-450 hepticos, a los que inhiben en mayor o menor mdicos asociados) y en las depresiones con intensas ideas suicidas.
medida (parece que citalopram y sertralina menos que los dems Tiene escasos efectos secundarios fsicos, lo que explica su utilidad en
ISRS) modicando al alza los niveles plasmticos de diversos frmacos, casos de intolerancia o contraindicacin para el uso de antidepresivos,
algunos potencialmente peligrosos por sus efectos cardiotxicos. en pacientes polimedicados o con patologa orgnica compleja y en
las psicosis graves durante el embarazo.
Antidepresivo Ventajas Desventajas
- Ecacia antidepresiva - Numerosos efectos Las guas clnicas llegan a decir que no existe ninguna contraindica-
demostrada secundarios cin absoluta para la TEC pero algunos libros mencionan el riesgo que
ADT - Precio - Letalidad en sobredosis supone utilizarla en casos de hipertensin intracraneal (pues durante
- Control de niveles - Posologa compleja cualquier crisis epilptica aumenta la produccin de lquido cefalorra-
plasmticos (imipramina) (ascenso, fraccionamiento)
qudeo y cabra la posibilidad de una herniacin cerebral). En general,
- Escasos efectos - Precio elevado las contraindicaciones dependen del procedimiento anestsico y cada
secundarios - Interacciones
ISRS - Escasa letalidad farmacocinticas vez son ms raras.
en sobredosis - Ecacia antidepresiva El principal efecto secundario es de tipo neuropsicolgico: la produccin
- Comodidad de uso menor? de una amnesia antergrada. Depende del estado cognitivo previo
(ms frecuente en ancianos), del nmero de sesiones recibidas, de
Pregunta 33. Ventajas y desventajas de los antidepresivos tricclicos y los la carga elctrica suministrada y de la disposicin de los electrodos.
inhibidores selectivos. Al cabo de unos meses NO existen secuelas del tratamiento (salvo
ciertas lagunas de memoria consecuencia del perodo de amnesia)
Pregunta 34.-R: 2 y el funcionamiento cognitivo de los pacientes es incluso mejor que
Los IMAO tenan un perl de tolerancia ligeramente mejor que los ADT, si hubieran recibido frmacos, pues la recuperacin de la depresin
pero dos importantes problemas prcticos: llega a ser ms completa.
La necesidad de seguir una dieta para evitar posibles crisis hiper- La mala fama de la TEC procede de la poca inicial, en la que se aplicaba
tensivas adrenrgicas secundarias a compuestos vasopresores sin anestesia (por estar poco desarrollados los procedimientos anest-
derivados de las aminas (tiramina). sicos) y en la que se usaba de forma indiscriminada para casi cualquier
Algunas interacciones farmacolgicas potencialmente peligrosas tipo de paciente psiquitrico.
por sus efectos noradrenrgicos (vasoconstrictores nasales) o sero- No hay que olvidar que fuera de la depresin mayor la TEC apenas tiene
toninrgicos (ISRS, meperidina). indicaciones; se puede usar en los episodios manacos, cuando son muy
graves o se muestran resistentes a los frmacos (algo poco frecuente);
Aunque la dieta no es complicada (bsicamente se trata de evitar todos los tambin se va a utilizar en episodios esquizofrnicos agudos resistentes a
alimentos que han sufrido procesos de fermentacin, como el queso, las antipsicticos, sobre todo si existen sntomas catatnicos o si hay clnica
bebidas alcohlicas fermentadas como el vino o la cerveza, los ahumados, depresiva asociada.

CTO Medicina C/Francisco Silvela, 106 28002 - Madrid Tfno. (0034) 91 782 43 30/33/34 E-mail: secretaria@ctomedicina.com www. ctomedicina.com 10
Comentarios de Test a distancia 1. vuelta
Psiquiatra
Pregunta 36.-R: 4 el EEG es que ya se ha provocado neurotoxicidad, que con frecuencia
El litio es el estabilizador de primera eleccin en la enfermedad bipolar. ser irreversible.
Su principal benecio es la reduccin del nmero de recadas (sobre
todo manacas) y tambin consigue atenuar la intensidad de las mismas, Pregunta 37.-R: 5
caso de que se produzcan. Los niveles de litio deben estar entre 0,4 y 1,5 mEq /l. Lgicamente, en
Ms discutible es su ecacia en los episodios agudos; como antidepre- los episodios agudos sern necesarias litemias ms altas (superiores a
sivo, el litio es muy poco potente y no es raro que se deban usar otros 1,0 mEq/l) que en los perodos de estabilidad (inferiores a 1,0 mEq/l); la
frmacos asociados (antidepresivos con muchas precauciones por el mayora de los pacientes van a estar un poco por arriba o por debajo de
riesgo de cambio de fase: lamotrigina, quetiapina) o recurrir al TEC; esta cifra (1,0 mEq/l), para no provocar una excesiva toxicidad o perder
como antimanaco, el litio es muy ecaz pero tiene un efecto retardado la ecacia.
(varias semanas) que puede resultar inaceptable para pacientes manacos
graves y, adems, no se dispone de otra presentacin aparte de la oral,
lo que condiciona su uso a la aceptacin del tratamiento por parte del
paciente (que no es lo habitual en las formas graves); por eso, en las
fases manacas graves se recurre al uso de frmacos por va parenteral
(antipsicticos, benzodiacepinas), para introducir el estabilizador en un
segundo momento. El litio es especialmente ms ecaz en los cuadros de
mana clsica (eufrica), disminuyendo su ecacia en los cuadros atpicos
(manas disfricas o irritables, fases mixtas, etc.), prerindose en estos
casos otros frmacos (valproico, carbamacepina, antipsicticos atpicos).
Adems, el litio es un medicamento con numerosos efectos secundarios,
hasta tal punto que si un paciente no reere ningn problema durante
el tratamiento, probablemente no lo est tomando. Para prevenir los
efectos adversos ms graves se realizan una serie de controles mdicos
antes y durante el tratamiento con litio; con diferencia, lo ms importante Pregunta 37. Mrgenes de las litemias.
ser el control de los niveles plasmticos de litio (litemia), pues guardan
una relacin bastante estrecha con la ecacia y la toxicidad. Si el episodio manaco o depresivo es leve, quizs baste con una ligera
correccin de la dosis del estabilizador, pero en los dems casos ser
Hemograma necesario aadir tratamiento especco; en los casos de mana grave la
Pruebas de funcin renal (creatinina, urea) falta de colaboracin del paciente suele obligar al tratamiento inicial con
Estudio inico (sodio, potasio, calcio) psicofrmacos parenterales (antipsicticos, benzodiacepinas), dejando
Pruebas de funcin tiroidea (TSH, T4 libre)
la adicin del estabilizador (oral) para ms adelante.
ECG
Test de embarazo (al inicio) Una vez conseguido el control de la recada se retirar el frmaco especco
Glucemia y cuerpos cetnicos (si se sospecha intolerancia a los hidratos de la fase en cuanto sea posible, para no inducir un cambio de polaridad.
de carbono) Cerca de 1/3 de los pacientes podrn estabilizarse a largo plazo con un
Pruebas de concentracin de la orina (si se sospecha diabetes inspida
solo frmaco, pero cerca de un 20% de los pacientes precisarn una pauta
nefrognica)
de medicacin compleja (con cuatro o ms frmacos).

Pregunta 36. Controles necesarios en el tratamiento con litio. Pregunta 38.-R: 4


Otro problema adicional del litio es la posibilidad de que los niveles
Dado que el litio se elimina por el rin, utilizando los mismos sistemas plasmticos se modiquen por factores que afectan la eliminacin renal,
de transporte que el sodio, ser necesario disponer de parmetros que sin que el paciente haya variado la dosis. Dado que el litio utiliza los
nos informen del estado de la funcin renal (creatinina, BUN). mismos sistemas de transporte renal que el sodio (se reabsorbe sobre
Uno de los efectos ms peligrosos es la alteracin de la conduccin todo en el tbulo proximal), cualquier circunstancia que aumente la
cardaca; las alteraciones en el ECG permiten controlar hasta qu punto reabsorcin de este ion (hipovolemia, deshidratacin, hiponatremia)
el litio produce problemas a este nivel (normalmente se van a limitar a producir un aumento de los niveles de litio, superando el umbral de
cambios en la onda T). toxicidad. Numerosos frmacos van a producir los mismos problemas,
El litio es un medicamento teratgeno; se ha relacionado con malforma- bien por sus efectos sobre la vascularizacin renal (AINE, inhibidores
ciones cardacas (anomala de Ebstein), por lo que antes de iniciar el trata- de la COX-2), bien por sus efectos sobre el tbulo renal (diurticos,
miento con litio en una mujer en edad frtil conviene asegurarse de que no IECA, ARA-2).
est embarazada e informarla de la necesidad de que adopte medidas de En teora podramos ver una disminucin de la litemia, con la consiguiente
anticoncepcin ecaces; ante cualquier sospecha de embarazo habr que prdida de ecacia, en las situaciones hidroelectrolticas inversas (hiper-
repetir la prueba diagnstica de gestacin pues, de conrmarse, se debera natremia, ingesta hdrica excesiva) y por algunos frmacos (metilxantinas,
interrumpir el tratamiento (nunca de forma brusca por el riesgo de recada). diurticos osmticos e inhibidores de la anhidrasa carbnica), pero son
Finalmente, resulta obligatorio el control de la funcin tiroidea mediante situaciones excepcionales.
la determinacin de la TSH para detectar problemas tiroideos (que en
ocasiones explican la resistencia al tratamiento). Otras determinaciones Pregunta 39.-R: 2
(glucemia, calcemia, pruebas de concentracin de la orina) dependern La ciclacin rpida es una complicacin extremadamente grave en la
de la evolucin y de la aparicin de efectos secundarios. evolucin del trastorno bipolar. El paciente pasa de una fase a otra casi
El electroencefalograma NO es una prueba necesaria para el control del sin estar unos das estable. Sucede con ms frecuencia en bipolares-2,
litio ni de ningn otro psicofrmaco; cuando aparecen alteraciones en muchas veces no diagnosticados de bipolares y tratados exclusivamente

CTO Medicina C/Francisco Silvela, 106 28002 - Madrid Tfno. (0034) 91 782 43 30/33/34 E-mail: secretaria@ctomedicina.com www. ctomedicina. 11
Comentarios de Test a distancia 1. vuelta
Psiquiatra
con antidepresivos. Por eso, es un poco ms frecente en mujeres (el es el nivel de desarrollo de la regin (p.ej., mejor en zonas rurales que
trastorno bipolar-2 es algo ms frecuente en el sexo femenino). en reas urbanas).
En ocasiones se detectan alteraciones tiroideas que necesitan ser Existe una estacionalidad referida al mes de nacimiento de los pacientes,
corregidas para frenar la ciclacin. En general no responden bien a encontrndose un mayor nmero de pacientes nacidos en los meses
la monoterapia con litio; quizs respondan algo mejor al tratamiento fros del ao, lo que se ha relacionado con diversos factores causales
con anticonvulsivos, pero lo ms habitual es utilizar como mnimo dos (infecciones vricas).
estabilizadores. Si algo nos recuerdan estos pacientes es la necesidad de Estadsticamente, los pacientes tienen un mayor riesgo de muerte tanto
extremar las precauciones al usar antidepresivos en pacientes bipolares. por causas no naturales (suicidio) como por causas naturales.

Pregunta 40.-R: 2 Pregunta 42.-R: 4


Como ya dijimos, el litio es el medicamento de primera eleccin en la En la actualidad, el diagnstico de esquizofrenia se basa en los siguientes
prevencin de recadas de la enfermedad bipolar pero no es la solucin criterios:
denitiva. Hay pacientes que no toleran sus efectos adversos, mientras 1) Presencia de unos sntomas caractersticos que se han dividido
que en otros est contraindicado (insuciencia renal, psoriasis, enfer- en positivos (los que predominan en las fases agudas o brotes) y
medad del nodo sinusal). Adems, hay un alto porcentaje de pacientes negativos (que son ms evidentes en las fases residuales).
(30-40%) que no responden bien a este medicamento: 2) Ausencia de otras enfermedades psiquitricas, enfermedades m-
Podemos predecir que no van a responder al litio: dicas, frmacos o txicos que los justiquen.
Las manas atpicas o disfricas (irritables). 3) Produccin de un deterioro en el funcionamiento habitual del pa-
Los episodios mixtos (que mezclan sntomas depresivos y manacos). ciente (acadmico, social, laboral).
Los cuadros manacos secundarios a enfermedades mdicas. 4) Una duracin determinada:
Los pacientes con ciclacin rpida. - La clasicacin DSM especica que debe ser superior a 6 meses,
comprendiendo los prdromos, el brote psictico (que debe
Algunos anticonvulsivos se han convertido en frmacos de referencia durar como mnimo un mes, salvo que se trate precozmente)
para el trastorno bipolar, sobre todo el cido valproico. Tambin juegan y la fase residual de recuperacin. Si la duracin es inferior a 6
un papel muy importante en el control de la enfermedad la carbamace- meses se diagnosticar de trastorno psictico breve si no dura
pina, su derivado oxcarbacepina o la lamotrigina (este ltimo para las ms de un mes o de trastorno esquizofreniforme si dura entre
fases depresivas). Los anticonvulsivos tienen menor toxicidad, lo que uno y 6 meses.
explica su utilizacin en casos mdicamente complejos o pacientes - La CIE slo acepta el diagnstico de trastorno psictico breve
con intolerancia al litio. Se controlan de una forma similar a su uso en cuando dura menos de un mes, llamando tambin esquizofrenia
la epilepsia, aunque no se ha demostrado que los niveles plasmticos a los trastornos esquizofreniformes de la DSM.
guarden relacin con su ecacia (s con la toxicidad). Otros anticon-
vulsivos (clonazepam, topiramato, gabapentina, pregabalina) tienen POSITIVOS NEGATIVOS
menor utilidad, aunque pueden usarse como frmacos coadyuvantes De novo, no presentes Prdida de una funcin
en casos concretos. Concepto
en la experiencia normal psicolgica normal
Los antipsicticos atpicos han ido demostrando ecacia en distintos Productivos, "psicticos",
aspectos de la enfermedad. Su accin antimanaca est aceptada con Sinnimos Decitarios, residuales
"activos"
claridad, existiendo datos sobre la ecacia antidepresiva de algunos de De curso breve, agudos Crnicos, estables
ellos (quetiapina) y sobre su capacidad de prevenir recadas. Fciles de identicar en el tiempo
y valorar Difciles de valorar
Caractersticas Gran acuerdo entre Discrepancias entre
diferentes entrevistadores entrevistadores
TRASTORNOS PSICTICOS. FRMACOS ANTIPSICTICOS Recuerdan a los de primer Recuerdan a los
rango de Schneider primarios de Bleuler
Pregunta 41.-R: 3 Escalas BPRS, PSE, SADS, etc. PANSS, SANS
La esquizofrenia es una enfermedad frecuente, afectando a cerca del
Alucinaciones Pobreza del lenguaje
1% de la poblacin en algn momento de su vida; teniendo en cuenta Delirios Aplanamiento afectivo
que tan slo 1/3 de los casos se pueden considerar de buen pronstico, Ejemplos Catatona Asociabilidad, anhedonia
con una recuperacin ms o menos completa, se puede calcular que en Conductas extraas (?) Dcit de atencin
Disgregacin (?) Afecto inapropiado (?)
Espaa hay cerca de 300.000 personas con la enfermedad.
Tiende a aparecer en la adolescencia o juventud (de los 15 a los 35 aos),
siendo excepcional su inicio en la infancia y en la vejez. Pregunta 42. Sntomas positivos y negativos de la esquizofrenia.
Afecta por igual a hombres y mujeres, si bien en las mujeres tiene un
inicio algo ms tardo (quizs por el efecto protector de los estrgenos), Pregunta 43.-R: 3
lo que determina un mejor pronstico. Desconocemos cules son las causas precisas de esta enfermedad, pero
No se han descrito variaciones signicativas de su incidencia en funcin de se maneja un modelo terico que incluye factores que aumentaran el
la clase social, la cultura o la raza; sin embargo, s se encuentran diferencias riesgo para padecerla (factores predisponentes o de vulnerabilidad)
en la prevalencia en determinadas regiones del mundo o determinados y otros que contribuiran a su aparicin en individuos predispuestos
grupos poblacionales que pueden explicarse por diferencias en el (factores precipitantes o desencadenantes).
pronstico y la mortalidad. As, la mayora de los pacientes tiene un nivel Dentro de los factores de vulnerabilidad que predisponen a sufrir es-
socioeconmico bajo, como consecuencia del deterioro que produce la quizofrenia destaca, sobre todo, la presencia de antecedentes familiares
enfermedad (hiptesis del descendimiento social); el pronstico en lo de esquizofrenia; la heredabilidad de la esquizofrenia explica el 80%
que se reere a la adaptacin social parece ms favorable cuanto menor de la varianza de su aparicin, si bien la penetrancia de la enfermedad

CTO Medicina C/Francisco Silvela, 106 28002 - Madrid Tfno. (0034) 91 782 43 30/33/34 E-mail: secretaria@ctomedicina.com www. ctomedicina.com 12
Comentarios de Test a distancia 1. vuelta
Psiquiatra
es incompleta y se implican numerosos genes dentro de un modelo En los delirios destac tanto su contenido (delirios de control e inuen-
extremadamente complejo. cia frente a los mucho ms frecuentes, pero ms inespeccos delirios
de perjuicio) como la forma de aparicin (percepciones delirantes sin
Otros factores que aumentan en menor grado la posibilidad de sufrir sentido, frente a las mucho ms frecuentes interpretaciones delirantes,
esquizofrenia son: ms o menos conectadas con la realidad).
Haber nacido en los meses fros del ao (se ha relacionado con Ejemplo de percepcin delirante: veo una persona apoyada en una
infecciones por el virus de la gripe durante el embarazo). farola de la calle, iluminada por su luz; esa luz quiere decir que soy
Antecedentes de problemas obsttricos (el sufrimiento fetal se el nuevo Mesas, y ese es el Demonio que viene a impedir que me
asociara con dao cerebral). manieste.
Enfermedades graves de la primera infancia (sobre todo las que Ejemplo de interpretacin delirante: veo una persona apoyada en
afectan directamente al sistema nervioso central, como meningo- una farola de la calle, iluminada por su luz; comienzo a pensar si no
encefalitis, epilepsia grave, traumatismos craneales severos). lleva mucho tiempo all y si no estar esperando a que salga para
hacerme dao.
Los factores desencadenantes NO producen esquizofrenia en personas
sin vulnerabilidad; su inuencia es ms evidente en las recadas de la Pregunta 46.-R: 3
enfermedad: En las fases agudas o brotes predominan los sntomas positivos y en las
Txicos (sobre todo alucingenos, cannabis y estimulantes como la fases residuales los sntomas negativos. El aplanamiento afectivo es un
cocana o las anfetaminas). sntoma negativo.
Estrs social (especialmente importante para explicar algunas recadas Vamos a detenernos un poco en el diagnstico diferencial de los sntomas
debidas al abandono del tratamiento). negativos; stos pueden deberse a diferentes causas, cada una de ellas
Enfermedades graves que afectan al sistema nervioso central. con una solucin especca:
En ocasiones, los sntomas se deben a que el enfermo est deprimido
La existencia de claros factores desencadenantes es un dato de mejor (es frecuente al principio de la enfermedad, cuando tienen todava
pronstico (siempre ser mejor que el cerebro se rompa como conse- conciencia de su cambio), pudiendo responder al tratamiento anti-
cuencia de una causa que lo haga espontneamente). depresivo.
Por otro lado, los frmacos antipsicticos tienen la capacidad de
Pregunta 44.-R: 4 producir sntomas extrapiramidales que mimetizan los sntomas
Los trastornos del pensamiento son, junto con las alteraciones de la per- negativos y responden a una reduccin o un cambio de la medi-
cepcin, los sntomas ms conocidos de la esquizofrenia; los podemos cacin.
dividir en trastornos del contenido del pensamiento (ideas delirantes) No hay que olvidar que la repercusin social de la enfermedad (aisla-
y trastornos del curso o la forma del pensamiento. miento, rechazo) conduce a muchos enfermos a una situacin similar
Dentro de las ideas delirantes, hablamos de ideas delirantes primarias a la descrita en los cuadros decitarios que mejora radicalmente al
cuando parecen surgir de forma autnoma, independientes del resto comenzar un tratamiento de tipo rehabilitador.
de fenmenos psicopatolgicos, siendo tpicas de la esquizofrenia y la Por ltimo, la persistencia de sntomas positivos intensos puede hacer
paranoia; las ideas delirantes secundarias (o ideas deliroides) proceden que algunos enfermos rechacen el contacto social o se muestren
de fenmenos de tipo afectivo (depresin, mana, angustia). retrados como consecuencia del miedo que tienen a ser daados
En lo referente a las alteraciones del curso y la forma podemos ver por los dems.
dicultades para dirigir el pensamiento hacia unas conclusiones lgi-
cas (tangencialidad), apareciendo con frecuencia prdidas de sentido Una vez controlados todos estos sntomas negativos secundarios nos
(descarrilamientos) que acaban por producir la desestructuracin del encontraramos con los sntomas negativos primarios, derivados real-
pensamiento (disgregacin) que puede recordar la incoherencia de los mente de la enfermedad.
pacientes con trastornos cognitivos graves; en otras ocasiones vamos
a encontrar un pensamiento muy empobrecido (alogia), quejndose
los pacientes de interrupciones en curso del pensamiento (bloqueos).

Pregunta 45.-R: 2
Kurt Schneider fue un psiquiatra que se plante facilitar el diagnstico
de la esquizofrenia mediante un estudio de aquellos sntomas cuyo
diagnstico fuera ms fcil, siendo adems sucientemente frecuentes y
razonablemente tpicos. Buscaba lo que hoy en da llamaramos sntomas Pregunta 46. Historia natural de la esquizofrenia.
de alto valor predictivo positivo, aunque se equivoc al creer que eran
frecuentes (aparecen en slo un tercio de los pacientes), por lo que su Pregunta 47.-R: 2
valor predictivo negativo es bajo. Los factores pronsticos de la esquizofrenia han sido preguntados en
Hay que reconocer que son altamente sugerentes de esquizofrenia, varias ocasiones; podemos dividirlos de una forma sencilla en:
pero NO patognomnicos. No olvidar que en la esquizofrenia son Factores previos al inicio de la enfermedad:
mucho ms frecuentes los sntomas negativos, que por desgracia - Sexo (mujeres mejor que varones).
son mucho ms inespeccos. - Antecedentes familiares (mejor si NO hay antecedentes de es-
Dentro de las alucinaciones destac las auditivas, en concreto aqullas quizofrenia).
en las que se escuchan voces humanas que hablan entre s del enfermo - Nivel de adaptacin previo (mejor cuanto mayor fuera).
y de su vida (no, por ejemplo, voces que insultan, que son con mucho - Personalidad previa (mejor si era normal).
las alucinaciones ms frecuentes en toda la psiquiatra). - Nivel de inteligencia previo (mejor si NO era bajo).

CTO Medicina C/Francisco Silvela, 106 28002 - Madrid Tfno. (0034) 91 782 43 30/33/34 E-mail: secretaria@ctomedicina.com www. ctomedicina. 13
Comentarios de Test a distancia 1. vuelta
Psiquiatra
Factores relacionados con el debut de la enfermedad: Delirios de celos (celotipia delirante).
- Edad de inicio (uno de los fundamentales; mejor cuanto ms tarda). Delirios de enfermedad (hipocondra delirante).
- Presencia de factores precipitantes (mejor si los hay). Delirios de grandeza.
- Presencia de sntomas depresivos o manacos (mejor si los hay). Delirios de enamoramiento.
- Presencia de sntomas confusionales (mejor si los hay).
- Inicio rpido (mejor que si el inicio es insidioso). Pregunta 50.-R: 3
- Patrn de sntomas (mejor si dominan los positivos). El mecanismo por el cual los pacientes paranoicos llegan al delirio es
- Rapidez a la hora de iniciar un tratamiento. la interpretacin delirante de la realidad; con frecuencia basan sus
delirios en hechos que realmente han sucedido, pero a los que dan
Factores evolutivos: un significado especial que no se aleja de lo razonable (a diferencia
- Nmero de recadas (fundamentalmente en los 5 o 10 primeros de los delirios esquizofrnicos, cuyos razonamientos pueden ser
aos). increbles y difciles de comprender). Psicolgicamente estos pa-
- Gravedad de los sntomas negativos (determinantes para el cientes recurren a la proyeccin como estrategia de afrontamiento
pronstico social y laboral). del estrs (mecanismo de defensa), al situar en el exterior la causa
- Resistencia al tratamiento de los sntomas positivos. de sus problemas (por ejemplo, si no encuentro algo es porque me
lo han quitado).
Pregunta 48.-R: 3 Clsicamente no se admite la existencia de alucinaciones en los tras-
En la prevencin de las recadas de la enfermedad esquizofrnica resulta tornos delirantes, aunque podemos ver alteraciones de la percepcin
fundamental tanto el control de los factores que favorecen las mismas (sobre todo ilusiones tctiles u olfativas) que pudieran confundirse con
(consumo de txicos, manejo del estrs) como el mantenimiento de un actividad alucinatoria.
tratamiento antipsictico a largo plazo; sin embargo, la adherencia al
tratamiento farmacolgico en estos pacientes es muy baja y los factores Pregunta 51.-R: 4
que inuyen en el abandono del mismo condicionan la probabilidad Los antipsicticos clsicos o tpicos ejercen diferentes acciones sobre
de recada: los sistemas de neurotransmisin cerebrales:
El perl de efectos secundarios del medicamento: para evitar el Poseen la capacidad de bloquear los receptores dopaminrgicos;
abandono por esta razn debemos individualizar el tratamiento el bloqueo D2 se ha relacionado tanto con su efecto antipsictico
segn las preferencias del paciente; teniendo en cuenta que los como con sus efectos secundarios extrapiramidales (por accin
efectos extrapiramidales son muy mal tolerados, por lo que los sobre la va nigroestriada) y endocrinos (hiperprolactinemia por
antipsicticos atpicos parecen tener una mejor aceptacin. su accin sobre la va tuberoinfundibular); adems se relaciona
La conciencia de enfermedad: puede mejorarse mediante el trabajo con su capacidad para inhibir el vmito y el hipo inducidos por
de psicoeducacin en la enfermedad, sobre todo a travs de terapias quimioterapia.
grupales. Los ms antiguos tienen importantes efectos antihistamnicos (que
La gravedad del estrs psicosocial sufrido por el paciente: en muchos producen somnolencia y aumento del apetito y del peso), efectos
casos es al ambiente familiar la principal fuente de tensin psicol- antiadrenrgicos (con hipotensin ortosttica como consecuencia) y
gica; en su da se cre el trmino expresin emocional elevada para efectos anticolinrgicos (produciendo el tpico sndrome muscarnico:
referirse a esas familias en las que se evidenciaban unos mecanismos sequedad de boca, estreimiento, visin cercana borrosa, retencin
patolgicos de adaptacin a la enfermedad; puede disminuirse esta urinaria, alteraciones de memoria).
tensin mediante tcnicas de intervencin familiar, el uso de recur-
sos como centros de da u hospitales de da y, en ltima instancia, Pregunta 52.-R: 3
mediante alternativas residenciales. Dentro de los antipsicticos tpicos o neurolpticos encontramos
diferencias en su potencia (capacidad de bloqueo dopaminrgico)
Pregunta 49.-R: 2 que pueden ser compensadas con un aumento de dosis; usando dosis
La paranoia o trastorno delirante es una enfermedad sorprendente y poco equivalentes la ecacia antipsictica es similar en todos ellos, pero al
frecuente en la que los pacientes tienen bsicamente un nico sntoma aumentar la dosis van a aparecer ms efectos sistmicos como con-
(un delirio), con una llamativa preservacin del resto del funcionamiento secuencia de los bloqueos colaterales descritos (menor potencia, ms
psicolgico y un escaso deterioro de su funcionamiento. dosis, mayor sedacin, ms hipotensin, ms acciones anticolinrgicas);
Aunque se pueden ver cuadros paranoicos de inicio agudo, en sin embargo, en los antipsicticos menos potentes la presencia de un
respuesta a un estrs identificable (como los descritos en presos bloqueo anticolinrgico signicativo va a disminuir la tasa de efectos
o inmigrantes), la paranoia clsica tiene un inicio insidioso, casi extrapiramidales (puesto que as en la va nigroestriada los dos neuro-
siempre entrelazado con una personalidad previa peculiar (pa- transmisores estarn bloqueados).
ranoide, demasiado sensible), de tal forma que cuando el delirio Las consecuencias de la hiperprolactinemia inducida por los antipsicti-
toma importancia el paciente suele encontrarse en la edad adulta, cos son bien conocidas: en mujeres veremos amenorrea, ginecomastia,
sufriendo consecuencias en los terrenos social, familiar o laboral galactorrea y disminucin de la libido, pero se han descrito casos de
segn los contenidos del delirio. hipogonadismo grave con repercusin en la masa sea de las pacientes;
Tiene un curso crnico, con una respuesta discreta del delirio al trata- en varones, la hiperprolactinemia produce disfunciones sexuales y dis-
miento; con la medicacin se va a buscar ms un control conductual minucin de la libido sobre todo. Dentro de los antipsicticos clsicos,
que una disolucin del delirio (encapsulacin). destaca la capacidad del sulpiride para inducir aumentos enormes de
Las clasicaciones actuales aceptan cinco formas de delirios crnicos la prolactina, incluso cuando se usa en dosis bajas. Para encontrar un
paranoicos: antipsictico que no modique la prolactina hay que irse al grupo de
Delirios de persecucin (los ms frecuentes, pero muy poco espec- antipsicticos atpicos, sobre todo la clozapina y frmacos anes (olan-
cos). zapina, quetiapina).

CTO Medicina C/Francisco Silvela, 106 28002 - Madrid Tfno. (0034) 91 782 43 30/33/34 E-mail: secretaria@ctomedicina.com www. ctomedicina.com 14
Comentarios de Test a distancia 1. vuelta
Psiquiatra
Ya existen algunas presentaciones parenterales de los antipsicticos
atpicos, si bien slo la risperidona de accin prolongada (consta) ha
logrado imponerse frente los antipsicticos depot tpicos.
Tienen un precio muy superior a los frmacos tradicionales, sin que
por el momento se haya podido demostrar en estudios de coste-
efectividad que compensen su precio con una reduccin de otros
costes de la enfermedad.

Pregunta 52. Relacin entre dosis


y potencia en los antipsicticos clsicos.

Pregunta 53.-R: 4
Los modernos antipsicticos atpicos se han convertido en los frmacos
de primera eleccin para el tratamiento de la mayora de los trastornos
psicticos. Se diferencian tericamente de los antipsicticos clsicos en
tres caractersticas:
Producen menos efectos secundarios extrapiramidales y neuroen-
docrinos (prolactina).
Tienen un mayor efecto antipsictico que los tpicos (tomando como
referencia el haloperidol).
Tienen cierto efecto sobre los sntomas negativos de la enfermedad
(accin de la que carecen los antipsicticos tpicos).
Pregunta 53. Evolucin de los antipsicticos.
La clozapina fue el primer antipsictico atpico y es el que cumple real-
mente con todas las caractersticas descritas, pero su asociacin con Pregunta 54.-R: 1
agranulocitosis ha hecho que pase a ser un frmaco de reserva, utilizado De todos los efectos secundarios que pueden presentar los antipsicticos,
cuando todos los dems fallan. los ms preguntados son los extrapiramidales; stos van cambiando en
funcin del tiempo:
Cmo consiguen ese perl farmacolgico diferencial? En las primeras horas o das podemos ver distonas agudas; son
Prcticamente todos combinan acciones antidopaminrgicas con contracciones sostenidas de grupos musculares que pueden resul-
acciones antiserotoninrgicas. tar dolorosas o peligrosas (distonas larngeas); son especialmente
Algunos de ellos tienen la capacidad de bloquear mltiples receptores frecuentes en varones, tanto ms cuanto ms jvenes.
(dopaminrgicos, serotoninrgicos, histaminrgicos, adrenrgicos, Al cabo de varios das o semanas aparecen dos tipos de efectos
colinrgicos), sin que se pueda asegurar en qu bloqueos radica su aparentemente opuestos: una reduccin de los movimientos en la
ecacia. forma de sndrome parkinsoniano o un aumento de la movilidad en
Otros deenden el bloqueo dopaminrgico selectivo de las vas la forma de inquietud (objetiva y subjetiva) o acatisia.
tericamente implicadas en los sntomas de la enfermedad (vas Cuando un paciente ha tomado medicacin antipsictica durante
mesolmbicas y vas mesocorticales), respetando la va nigroestriada, meses o aos puede presentar movimientos de tipo coreoatetsico
responsable de los efectos extrapiramidales. (los ms frecuentes de la musculatura faciobucolingual) denominados
En algunos son las caractersticas del bloqueo dopaminrgico (efecto discinesias tardas; en raras ocasiones afectan a msculos del tronco
de agonista parcial, constante de disociacin del receptor) las que o las extremidades produciendo una seria limitacin en el paciente;
parecen determinar la diferencia. su respuesta al tratamiento es pobre, pudiendo considerarse casi
irreversibles.
Sin embargo, estos frmacos no estn exentos de problemas. La clo-
zapina, la olanzapina y la quetiapina pueden producir un aumento Cada efecto secundario tiene un tratamiento especco:
del apetito y del peso muy llamativos, adems de efectos sobre la Anticolinrgicos para las distonas agudas (por va parenteral) y el
tensin arterial y somnolencia; el aumento de peso se ha relacionado parkinsonismo (por va oral): biperideno, trihexifenidilo.
con alteraciones metablicas (diabetes mellitus tipo 2, dislipemias) Betabloqueantes (propranolol) o benzodiacepinas (lorazepam) para
y aumento de la morbimortalidad cardiovascular. La risperidona la acatisia.
produce efectos extrapiramidales a dosis medias y aumenta los Deplecionantes dopaminrgicos (tetrabenacina) para las discinesias
niveles de prolactina incluso en dosis bajas, con las consecuencias tardas graves.
endocrinas conocidas. Amantadina para el parkinsonismo (no es habitual en Espaa).

CTO Medicina C/Francisco Silvela, 106 28002 - Madrid Tfno. (0034) 91 782 43 30/33/34 E-mail: secretaria@ctomedicina.com www. ctomedicina. 15
Comentarios de Test a distancia 1. vuelta
Psiquiatra
AGUDOS SUBAGUDOS TARDOS

Inicio Das Semanas Aos

Tipo Distonas Parkinsonismo Acatisia Discinesias

Temblor Corea facial


Crisis oculgiras
Clnica Acinesia Inquietud Distonas focales
Torticolis
Rigidez 15% graves

Ancianos
Jvenes
AP tpicos incisivos Mujeres
Varones
Factores de riesgo Dosis altas Dao cerebral
AP tpicos incisivos
Dao cerebral Trastornos afectivos
Dosis altas
Anticolinrgicos

Anticolinrgicos Anticolinrgicos orales BZD Clozapina


Tratamiento
parenterales (biperideno) Amantadina Betabloqueantes Tetrabenacina

Pregunta 54. Efectos extrapiramidales de los antipsicticos.

Un efecto secundario especialmente grave y con componente extrapi- en gran medida el pronstico sociolaboral de la enfermedad. De aqu
ramidal es el sndrome neurolptico maligno; en l se combinan: que en los ltimos aos se est hablando de nuevos problemas en el
Elementos extrapiramidales (rigidez extrema). manejo clnico de este tipo de pacientes, como el sndrome de la puerta
Hipertermia. giratoria (reingresos repetidos de los mismos pacientes por la ausencia
Alteraciones vegetativas (sudoracin, labilidad de la tensin arterial de recursos externos al hospital) o el problema de los pacientes jvenes
o la frecuencia cardaca). crnicos, especialmente grave en las grandes ciudades. Especial trascen-
Sntomas confusionales. dencia tiene la alta prevalencia de consumo de txicos en este grupo
de pacientes (la llamada patologa dual), dado que las sustancias de
Tiene una importante mortalidad cuando se deja sin tratamiento, fundamen- abuso tienden a exacerbar los sntomas de su enfermedad y complican
talmente por las repercusiones renales de la necrosis muscular masiva (por la enormemente el tratamiento farmacolgico.
rigidez y la hipertermia), siendo bsico su diagnstico precoz (analticamente Las terapias dinmicas (derivadas del psicoanlisis) carecen en la actuali-
veremos leucocitosis, aumento de la CPK y de otras enzimas musculares, dad de indicacin en este tipo de pacientes, puesto que precisan de una
hiperfosfatemia, hiperpotasemia) y su tratamiento inmediato; se suele capacidad de introspeccin importante y se enfrentan a una enfermedad
indicar el ingreso del paciente en una UCI para un control mdico estricto, con condicionantes neurobiolgicos evidentes.
precisando de una serie de medidas fsicas para frenar la hipertermia y de
determinados frmacos (dantroleno [relajante muscular], bromocriptina
[agonista dopaminrgico]) para revertir los sntomas principales. TRASTORNOS RELACIONADOS CON SUSTANCIAS

Pregunta 55.-R: 3 Pregunta 57.-R: 5


Enlazando con el comentario de la pregunta anterior, tenemos aqu un El alcohol es un depresor del sistema nervioso central y sus efectos de-
paciente que, tras 15 das de tratamiento con un antipsictico, comienza penden de la dosis ingerida, la capacidad de metabolizarlo (tolerancia
a presentar inquietud. Esta inquietud inducida por los antipsicticos es farmacocintica) y la adaptacin cerebral al consumo crnico (tolerancia
probablemente el sntoma extrapiramidal ms frecuente derivado de farmacodinmica). En general, encontramos una buena correlacin entre
estos frmacos y resulta intensamente desagradable. Recibe el nom- la alcoholemia y los efectos clnicos, de tal forma que si una persona
bre de acatisia (inquietud en griego) y supone una frecuente causa tiene una alcoholemia elevada y no maniesta signos de intoxicacin
de abandono del tratamiento. La actitud ante este cuadro consiste en podemos asegurar que tiene una dependencia del alcohol.
reducir la dosis del antipsictico y administrar un frmaco corrector (un De forma inversa vemos pacientes que con alcoholemias muy bajas
betabloqueante o una benzodiacepina), pudindose proceder al cambio sufren graves alteraciones conductuales (agresividad, agitacin); son las
de antipsictico en el caso de que nada funcionara. llamadas intoxicaciones idiosincrsicas o borracheras patolgicas, y su
origen parece ser la mayor sensibilidad de algunas personas (ancianos,
Pregunta 56.-R: 4 nios, pacientes con dao cerebral) a las sustancias sedantes (alcohol,
Cuando hablamos del tratamiento de la enfermedad no debemos benzodiacepinas, barbitricos). De hecho, algunos alcohlicos crnicos
pensar que todo se reduce al uso de frmacos; en ese caso bastara con desarrollan con los aos una tolerancia inversa como consecuencia del
asegurarnos el cumplimiento (con formas intramusculares depot) para dao cerebral y heptico debido al alcohol.
controlar la enfermedad. Cuando se ingiere una cantidad masiva de alcohol puede producirse la
Tan importante como el tratamiento farmacolgico van a ser todas las afectacin del tronco del encfalo, apareciendo somnolencia y llegando
medidas psicosociales destinadas a mejorar la adherencia al tratamiento en ocasiones a la depresin cardiorrespiratoria. En general, el tratamiento
(psicoeducacin en la enfermedad, terapias e intervenciones familiares) de las intoxicaciones alcohlicas es puramente sintomtico, evitando las
y a la rehabilitacin y reinsercin social de los pacientes (tcnicas de complicaciones comunes a cualquier coma (neumonas por aspiracin,
modicacin de conducta, tcnicas de afrontamiento del estrs). sndromes compartimentales por decbito prolongado, hipotermia por
La disponibilidad y gestin de los llamados recursos intermedios exposicin al aire libre), pero una caracterstica diferencial del coma et-
(hospitales de da, centros de da, centros de rehabilitacin psicosocial, lico es la aparicin de hipoglucemia como consecuencia de la inhibicin
centros de rehabilitacin laboral, recursos residenciales) condicionar de la gluconeognesis y la glucogenlisis; por eso es frecuente que a

CTO Medicina C/Francisco Silvela, 106 28002 - Madrid Tfno. (0034) 91 782 43 30/33/34 E-mail: secretaria@ctomedicina.com www. ctomedicina.com 16
Comentarios de Test a distancia 1. vuelta
Psiquiatra
estos pacientes se les pauten soluciones glucosadas, con la precaucin mientras que el segundo no suele poner en riesgo la vida del paciente
de poner previamente tiamina para no precipitar una encefalopata de y se origina habitualmente a partir de un consumo muy elevado.
Wernicke; en casos excepcionales se puede recurrir a la dilisis para eli- Otras diferencias entre ambas entidades pueden observarse en la tabla
minar cantidades extremas de alcohol y, sobre todo, cuando el paciente siguiente.
ha ingerido alcoholes no etlicos, mucho ms neurotxicos.
DELIRIUM TREMENS ALUCINOSIS ALCOHLICA
Desencadenante Abstinencia brusca Consumo elevado
Alteracin
S (delirium) No (conciencia clara)
de la conciencia
Visuales (microzoopsias)
Alucinaciones Escenogrcas Auditivas (insultos)
Inducibles
Raro (secundario
Delirio Ocupacional
a las alucinaciones)
Alteraciones
Frecuentes No
somticas
Mortalidad Alta sin tratamiento Rara
Asegurar ctes. vitales
Pregunta 57. Efectos siolgicos de las intoxicaciones BZD, clormetiazol Cese del consumo
y las abstinencias. Tratamiento Evitar NL Haloperidol
Si convulsiones: Mg Prolaxis de la abstinencia
Suplementos vitamnicos
Pregunta 58.-R: 3
Una de las complicaciones ms graves del alcoholismo es el sndrome de Tabla 59 Diferencias entre delirium tremens y alucinosis alcohlica.
abstinencia severo o delirium tremens. Asocia una importante mortalidad
cuando se deja sin tratamiento, fundamentalmente por las complicacio- Pregunta 60.-R: 2
nes hidroelectrolticas secundarias a los vmitos, la sudoracin profusa o En la prevencin de recadas del alcoholismo tenemos dos enfoques
la diarrea, que afectan de una forma especialmente grave a un paciente bien diferentes.
que suele tener problemas derivados del alcoholismo en diversos rganos Por un lado, el modelo tradicional se centra en el tratamiento psi-
y sistemas (hepatopata, cardiopata, pancreatitis, etc.). colgico con sus diversas variantes (individual, de pareja, familiar,
Para que se produzca un sndrome de abstinencia tiene que existir grupal), en el que son muy conocidos los grupos de autoayuda. Se
tolerancia, y sta se adquiere como consecuencia del consumo crnico propone con frecuencia para estos pacientes el uso de interdictores
de cantidades elevadas de alcohol; parece que en su fisiopatologa (disulram, cianamida), frmacos que bloquean el metabolismo del
intervienen mecanismos gabargicos y glutamatrgicos, lo que alcohol en uno de sus pasos intermedios (aldehdo deshidrogenasa),
quizs explique la elevada incidencia de convulsiones generalizadas provocando la acumulacin de acetaldehdo, el cual desencadena
tnico-clnicas. una reaccin histaminrgica. La base del tratamiento es conductual;
se trata de disminuir una conducta problemtica mediante el miedo
a una consecuencia negativa (castigo); por tanto, es necesario que el
CDT paciente est al tanto de su tratamiento, siendo necesario que rme
Buena sensibilidad y especicidad
Transferrina deciente un consentimiento informado.
Buena rapidez de cambio (2 semanas)
en carbohidratos
Precio elevado En los ltimos aos el concepto de craving ha ido ganando fuerza en el
(desialotransferrina)
tratamiento de las drogodependencias. El tipo ms habitual de craving
GGT Buena sensibilidad, baja especicidad
Gamma glutamil Rapidez de cambio moderada (6 semanas)
es el deseo que aparece condicionado a estmulos ambientales que
transpeptidasa Bajo precio se relacionan con el consumo; se propuso el uso de acamprosato (un
VCM Baja sensibilidad, mejor especicidad modulador gabargico y glutamatrgico) para este problema pero su
Volumen corpuscular Muy lento en cambiar (120 das) ecacia es muy discutible.
medio de los hemates Bajo precio Por otro lado, algunos pacientes cuentan una llamativa prdida de
Cociente GOT/GPT control tras el consumo de una pequea cantidad del txico; en estos
Sirve para liar una hepatitis (alcohlica > 2)
Relacin entre casos la naltrexona (antagonista opioide) ofrece una ecacia clnicamente
No es til si no hay hepatitis
las transaminasas signicativa, siendo el frmaco ms utilizado en el tratamiento a largo
Triglicridos, cido rico Poco sensibles y poco especcos plazo del alcoholismo.
Marcadores inespeccos de malnutricin,
Niveles de B12 o flico
no de alcoholismo Pregunta 61.-R: 4
En la dependencia de la herona, el tratamiento farmacolgico es ms
Pregunta 58. Marcadores del consumo excesivo de alcohol. sencillo al existir frmacos especcos para el sistema opioide endgeno,
tanto agonistas como antagonistas.
Pregunta 59.-R: 4 En la intoxicacin el exceso de actividad de los opiceos produce una
En este caso se nos plantea el diagnstico diferencial entre dos cuadros disminucin del nivel de conciencia, que asocia depresin respiratoria
tpicos del paciente alcohlico crnico: el delirium tremens y la alucinosis en los casos graves; la naloxona es el antagonista utilizado en estos
alcohlica. Ambos se caracterizan por la presencia de alucinaciones en un casos por va parenteral; su corta vida media (menos de 4 horas) hace
paciente dependiente del alcohol. Sin embargo, el primero es un cuadro necesaria la administracin consecutiva de varias dosis para revertir por
grave (y potencialmente mortal) desencadenado por la abstinencia, completo los efectos del txico.

CTO Medicina C/Francisco Silvela, 106 28002 - Madrid Tfno. (0034) 91 782 43 30/33/34 E-mail: secretaria@ctomedicina.com www. ctomedicina. 17
Comentarios de Test a distancia 1. vuelta
Psiquiatra
Durante la abstinencia se aprecian sntomas de dcit de actividad de Dado que la accin fundamental de esta sustancia es catecolaminrgica
los opiceos (mialgias, diarrea) y sntomas derivados de la hiperactividad (dopaminrgica, sobre todo) muchos de los frmacos utilizados actan
adrenrgica (HTA, taquicardia, midriasis, temblor). Se puede tratar con en estos sistemas.
frmacos opiceos (metadona, codena) o con frmacos que reduzca la El uso de antidepresivos (tricclicos, inhibidores selectivos, etc.) se basa
actividad adrenrgica (clonidina, guanfacina). en que aumentan el tono catecolaminrgico y alivian la sintomatologa
depresiva asociada al cese del consumo. Dado que el consumo crnico
S. ABSTINENCIA / obliga a una adaptacin de los sistemas dopaminrgicos cerebrales se
INTOXICACIN
DESINTOXICACIN propone el uso de diversos frmacos dopaminrgicos (desde la L-dopa
Abandono del consumo a los agonistas ms modernos) para aliviar algunos sntomas (disforia,
Sobredosis accidental
Administracin de sustancias craving). Finalmente se plantea la utilidad de algunos anticonvulsivos a
Causas antagonistas, agonistas/ la hora de reducir el deseo por consumir.
Intento de suicidio
antagonistas o agonistas
parciales
BENZODIACEPINAS de entrada)
MIOSIS MIDRIASIS EXCESO DE NA/A:
Depresin Abandono de hiperactividad Vasoconstriccin CONTROL EN UCI segn
Clnica cardiorrespiratoria adrenrgica (diarrea, crisis HTA hemorragias complicaciones)
Alteracin del nivel rinorrea...) isquemia cardaca... arritmias Monitorizacin ECG y TA
de conciencia Deseo de consumir isquemia cerebral... epilepsia NO usar betabloqueantes (riesgo
2 opciones: de agravamiento paradjico)
a) Sustitutivo: metadona en la EXCESO DE DA/A:
dosis equivalente al consumo Antipsicticos?
Delirios, alucinaciones
y reduccin gradual en 5-19 Riesgo de convulsiones
Tratamiento NALOXONA i.v. Agitacin
das (de ELECCIN)
b) Sintomtico: disminuir
hiperactividad simptica Pregunta 63. Tratamiento de la intoxicacin por cocana.
Agonistas -2 (clonidina)

Pregunta 61. Intoxicacin y desintoxicacin de opiceos.


TRASTORNOS COGNOSCITIVOS: DELIRIUM, DEMENCIAS
Y AMNESIAS
Pregunta 62.-R: 2
La abstinencia de la herona es un cuadro clnico aparatoso pero sin
gravedad; por tanto, y a diferencia de la abstinencia del alcohol, NO es Pregunta 64.-R: 4
una urgencia mdica; los pacientes no deben recibir tratamiento de Este paciente sufre un delirium o sndrome confusional agudo; esta
forma puntual en un servicio de urgencias, salvo que deban quedar alteracin del estado mental es el cuadro psiquitrico ms frecuente en
ingresados por otro motivo; la actitud ms correcta es derivarles a cen- enfermos ingresados en hospitales generales, como consecuencia de
tros especializados para que inicien un tratamiento reglado destinado los procesos mdicos, las intervenciones quirrgicas o los tratamientos
no slo a la abstinencia sino a la prevencin de recadas. farmacolgicos; el sustrato siopatolgico es una encefalopata aguda
Para favorecer el mantenimiento de la abstinencia se han probado cuyo origen pueden ser problemas originados en el cerebro o fuera de l
diferentes frmacos, aunque existen programas de tratamiento en los (procesos metablicos, txicos, frmacos, etc.). El sntoma inicial del delirium
que no se utiliza ayuda farmacolgica alguna. es una alteracin de la atencin y la concentracin, que con frecuencia
La naltrexona (antagonista opioide) puede bloquear los efectos de una conducen a la desorientacin (primero en el tiempo, ms adelante en
dosis puntual de herona, evitando el refuerzo positivo del consumo; el espacio); es frecuente asistir a grandes uctuaciones de la clnica, en
sin embargo, este bloqueo es competitivo y puede ser revertido con ocasiones relacionadas con factores externos (p.ej., empeoran al llegar la
una dosis sucientemente alta de herona, por lo que el paciente noche, pues la menor iluminacin hace que interpreten peor el entorno).
debe comprometerse a no intentar vencerlo (se le pedir que d su
consentimiento informado), pues se provocara una intoxicacin grave Pregunta 65.-R: 4
al haber perdido la tolerancia; por este motivo solamente se propone Se describen tres formas de delirium (agitado, estuporoso y mixto) en
para pacientes altamente motivados, sin caractersticas impulsivas y sin funcin del patrn de comportamiento. En realidad, en un mismo enfermo
problemas mdicos o psiquitricos asociados. o en un mismo proceso patolgico lo ms frecuente es la forma mixta.
La metadona (agonista opioide) consigue disminuir de forma ms Hay dos aspectos clnicamente importantes del delirium. En primer lugar,
ecaz las complicaciones legales y mdicas del consumo de herona, la AGITACIN de algunos delirium puede suponer un riesgo para la salud
permitiendo la progresiva normalizacin del paciente en todas las del paciente y de las personas que le rodean; por eso llaman al psiquiatra,
esferas afectadas por la drogodependencia; a dosis elevadas parece como experto en sedar enfermos agitados, para elegir el frmaco que
tener adems un efecto anticraving. Se plantea como un tratamiento a controle el comportamiento agresivo, sin empeorar el estado fsico.
largo plazo, pudiendo necesitarse aos hasta que se pueda plantear su El haloperidol es el frmaco de eleccin para el control de cualquier cua-
retirada; es el tratamiento recomendado para los pacientes refractarios dro de agitacin, pues carece de efectos cardiovasculares y respiratorios;
a otros tratamientos, los heroinmanos con problemas psiquitricos o su margen de seguridad es muy elevado, y con esas dosis curiosamente
mdicos graves, las embarazadas y los politoxicmanos. no se producen los temidos efectos extrapiramidales de estos frmacos.
Los antipsicticos ms sedantes (clorpromacina, levomepromacina, zu-
Pregunta 63.-R: 3 clopentixol) tienen el riesgo de producir una hipotensin grave y tienen
En el tratamiento de la dependencia de la cocana, las opciones de importantes efectos anticolinrgicos que agravaran la confusin mental.
apoyo farmacolgico son mucho ms limitadas y de hecho ningn me- Los antipsicticos atpicos se estn usando cada vez ms para estos cuadros
dicamento ha demostrado una ecacia signicativa a la hora de reducir de agitacin, si bien, cuando se hace precisa la va intramuscular (lo que
el riesgo de recada. no es nada raro) el haloperidol se convierte en casi la nica alternativa.

CTO Medicina C/Francisco Silvela, 106 28002 - Madrid Tfno. (0034) 91 782 43 30/33/34 E-mail: secretaria@ctomedicina.com www. ctomedicina.com 18
Comentarios de Test a distancia 1. vuelta
Psiquiatra

Pregunta 65. Caractersticas clnicas del delirium (sndrome confusional agudo).

Las benzodiacepinas a dosis elevadas pueden empeorar el estado mental cia REM y en la segunda se acorta con frecuencia; sin embargo, es una
y producir depresin respiratoria, sobre todo si el paciente est tomando prueba compleja y con niveles de sensibilidad y especicidad bajos, por
otros frmacos con efectos sedantes, sin embargo, son el tratamiento lo que no se suele utilizar.
de eleccin en los sndromes de abstinencia alcohlica grave (delirium
tremens), pues corrigen de forma especca la siopatologa del sndrome. Pregunta 68.-R: 3

Pregunta 66.-R: 3 PSEUDODEMENCIA


DEMENCIA
No hay que olvidar que todo delirium tiene una CAUSA que conviene DEPRESIVA
aclarar cuanto antes, pues implica una morbimortalidad notable; por Curso
eso realizamos diferentes pruebas para diagnosticar el origen del mis- Inicio Insidioso Agudo
mo (anlisis de sangre [hemograma, bioqumica completa, serologas,
Progresin Lenta Rpida
hemocultivos, etc.] y de orina, ECG, EEG, pruebas de imagen [Rx de
Duracin Larga Breve
trax, TC], puncin lumbar). El test de Rorscharch (test de las manchas
de tinta) es una prueba psicolgica proyectiva que explora los aspectos H personal Normal Depresin
inconscientes de la personalidad; es totalmente absurdo aplicarlo a estos H familiar Demencia Tr. afectivos
pacientes, con una alteracin grave de su nivel de conciencia. Clnica
Actitud ante la enfermedad Ocultacin Exageracin
Pregunta 67.-R: 4
Congruencia entre conducta
Cuando la combinacin de fallos de memoria, desorientacin, disnomia S No
y deterioro
y dispraxias, y la demostracin de alteraciones en la exploracin neurop-
Fluctuaciones clnicas No S
sicolgica elemental (MEC o Minimental) hace que sospechemos una
Empeoramiento nocturno Frecuente Raro
posible demencia, debemos seguir un sistema de diagnstico diferencial
para descartar posibles enfermedades que tienen un carcter reversible Humor Depresivo Lbil, indiferente
o, al menos, algo ms benigno que las demencias degenerativas. Inters social Adecuado Prdida precoz
El primer paso es descartar que no exista una causa externa que est Exploracin
inuyendo en el funcionamiento cerebral hasta el punto de producir Atencin y concentracin Afectadas Conservadas
una disfuncin cognitiva grave: intoxicaciones crnicas con frmacos
Respuestas tpicas Aproximadas "No s"
sedantes, abuso de txicos, alteraciones metablicas (insuciencia
Dcit de memoria Reciente > remota Reciente=remota
renal, hipotiroidismo, encefalopata heptica), anemias graves, lesiones
ocupantes de espacio intracraneal (tumores, quistes, abscesos), enferme- Lagunas mnsicas Raras Frecuentes
dades infecciosas o inamatorias con tratamiento ecaz (slis, esclerosis Esfuerzo en los test S No
mltiple), dcit vitamnicos, etc. Respuesta en los test Congruente Variable
Por tanto, al paciente con sospecha de demencia deben realizrsele prue- Respuesta a la privacin
bas analticas (hemograma, bioqumica plasmtica completa, serologas Empeoramiento Mejora
de sueo
[slis, VIH], sistemtico de orina, estudio funcional tiroideo, niveles de Cogniciones
Respuesta al amobarbital Confusin
B12 y flico), ECG y pruebas de imagen (Rx trax, TC), para descartar las depresivas
principales causas de demencia reversibles o tratables.
La polisomnografa, sin embargo, slo puede ayudar a diferenciar la Pregunta 68. Diferencias entre demencia
demencia de la depresin, pues en la primera puede aumentar la laten- y depresin.

CTO Medicina C/Francisco Silvela, 106 28002 - Madrid Tfno. (0034) 91 782 43 30/33/34 E-mail: secretaria@ctomedicina.com www. ctomedicina. 19
Comentarios de Test a distancia 1. vuelta
Psiquiatra
OTROS TRASTORNOS MENTALES: PERSONALIDAD, En los trastornos alimentarios fundamentales (anorexia y bulimia ner-
ALIMENTACIN, SUEO, INFANTILES viosa) el sntoma central es el mismo: una preocupacin PATOLGICA
por la posibilidad de engordar. Este miedo condiciona su relacin con
Pregunta 69.-R: 4 la comida, a la que ven ms como una enemiga que como un medio
El trastorno lmite de la personalidad es probablemente el que ms com- necesario para conseguir una actividad normal. A qu se debe ese
plicaciones psiquitricas produce, el que ms medicacin va a recibir, el miedo? Muchas veces no llegamos a saberlo, pero subyacen una serie
que ms veces va a acudir a Urgencias, el que ms veces va a ingresar y de pensamientos distorsionados y sobrevalorados (pero NO delirantes)
el que mayor riesgo de suicidio tiene. acerca de las consecuencias del aumento de peso en sus vidas. No es raro
En l se mezclan sntomas de todas las lneas de la psicopatologa encontrar adems distorsiones en la percepcin de su esquema corporal,
(ansiosos, emocionales, psicticos, alimentacin, impulsividad, abuso pero algunas pacientes, reconociendo que estn delgadas, no soportan la
de sustancias, etc.), en general de una intensidad importante pero de idea de recuperar peso o no son capaces de normalizar su alimentacin.
una duracin breve, y, casi siempre, en respuesta a alguna circunstancia Cuando este trastorno aparece en una chica con una alta capacidad de
vivida como frustrante. Probablemente por sto se trate del trastorno de autocontrol (generalmente con rasgos obsesivos de personalidad) vere-
la personalidad que ms puede responder al tratamiento farmacolgico mos cmo va a ser capaz de seguir una dieta restrictiva de forma estricta,
(antidepresivos, antipsicticos, estabilizadores), teniendo el cuidado de aguantando su deseo de comer (la palabra anorexia sugiere la falta de
elegir frmacos con un elevado nivel de seguridad en el caso de intoxi- apetito, lo cual no es cierto hasta que no se alcanza la emaciacin); como
cacin (no es raro que estos pacientes protagonicen autolesiones de consecuencia de esa dieta se produce una notable prdida de peso (se
carcter impulsivo). Se ha relacionado este trastorno con el antecedente considera importante una prdida del 15% del peso esperable para su
de abusos sexuales en la infancia, si bien este dato es muy cuestionado. edad y talla o un IMC inferior a 17,5); la desnutricin acarrea numerosas
complicaciones fsicas, entre las que destaca la amenorrea. Es lo que
Pregunta 70.-R: 1 denominamos anorexia nerviosa.
Los trastornos de personalidad aumentan el riesgo de presentar tras- Cuando el trastorno afecta a chicas con un menor autocontrol (con rasgos
tornos mentales, existiendo algunas relaciones especialmente claras: ms impulsivos) la dieta va a verse interrumpida por atracones bulmicos
Los esquizotpicos sufren con frecuencia episodios psicticos breves; en los que se ingiere una enorme cantidad de alimento en un tiempo
en ellos encontramos una elevada frecuencia de antecedentes fami- llamativamente corto, con una sensacin de prdida de control; para
liares de esquizofrenia; en algunas clasicaciones se considera a la compensar los atracones aparecen una serie de conductas destinadas
esquizotipia una forma latente de esquizofrenia pues sus sntomas a favorecer la prdida de peso, que van desde las conductas purgantes
recuerdan en ocasiones a los de esta enfermedad. (vmitos autoprovocados, abuso de laxantes, uso de diurticos) a otras
Los esquizoides tambin se relacionan con las enfermedades psi- menos agresivas (alternancia de ayuno y atracones, ejercicio compulsivo).
cticas, siendo la personalidad anormal que con ms frecuencia se Lo llamaremos bulimia nerviosa.
relaciona con la aparicin de esquizofrenia; de hecho, sus rasgos Sin embargo, vemos tambin algunas formas mixtas; lo ms frecuente es
se solapan con los sntomas negativos de la esquizofrenia en sus que con la cronicacin de una anorexia nerviosa las pacientes comiencen
formas simple o residual (pero a diferencia de esos pacientes, los a presentar conductas purgantes SIN tener atracones, para no absorber
esquizoides siempre han sido as). lo poco que coman (anorexia restrictiva-purgativa). Adems, con el paso
Los paranoides se relacionan muy fuertemente con los trastornos deli- de los aos, las pacientes con anorexia llegan a perder el control sobre su
rantes, ya sean agudos (psicosis reactivas breves) o crnicos (paranoia). alimentacin y comienzan a presentar atracones (lo que algunos llaman
bulimarexia). Ms raro es que una chica con bulimia llegue a enganchar
Pregunta 71.-R: 3 un perodo sucientemente largo de autocontrol como para que pierda
peso de forma signicativa.

Pregunta 72.-R: 4
Los atracones bulmicos guardan una estrecha relacin con una baja
capacidad de autocontrol y los rasgos impulsivos de personalidad; por
eso no es raro encontrar en las pacientes bulmicas una mayor frecuencia
de otros comportamientos impulsivos (abuso de drogas, autolesiones,
sntomas depresivos, promiscuidad, etc.), asocindose algunos casos al
trastorno lmite de la personalidad.
Suelen ser ms frecuentes por la noche (cuando es menos probable que
sean descubiertas) y tras un perodo de ayuno. Los alimentos deben ser
fciles de tragar y no es raro que se seleccionen sobre todo dulces (la ur-
gencia del atracn no hace posible la seleccin de alimentos hipocalricos).
De todas las conductas alimentarias anormales, slo los atracones son sus-
ceptibles de cierto tratamiento farmacolgico; la medicacin de eleccin
son los ISRS, utilizndose dosis mucho ms altas que las habituales para
una depresin. El topiramato se ha puesto de moda como alternativa a
los ISRS para regular la impulsividad relacionada con la comida.
En general, el tratamiento de las pacientes con trastornos de la conducta
alimentaria es multidisciplinar, implicndose especialistas en psiquiatra,
en endocrinologa y nutricin (el bajo peso es la principal causa de in-
greso), en modicacin de conducta (para ayudar en el autocontrol de
Pregunta 71. Esquema de los trastornos de la conducta alimentaria. los atracones y los vmitos) y en psicoterapia (para explorar los motivos

CTO Medicina C/Francisco Silvela, 106 28002 - Madrid Tfno. (0034) 91 782 43 30/33/34 E-mail: secretaria@ctomedicina.com www. ctomedicina.com 20
Comentarios de Test a distancia 1. vuelta
Psiquiatra
SUEO
VIGILIA ACTIVA RELAJACIN No REM 75% (SUEO SINCRONIZADO) REM 25%
(OJOS ABIERTOS) (OJOS CERRADOS) (SUEO
Fase 1 (5%) Fase 2 (45%) Fase 3 (12%) Fase 4 (13%) DESINCRONIZADO)
Theta (4-8 Hz) Sueo "delta" (<4Hz) Beta (>12 Hz)
Beta (>12 Hz) Alfa
EEG Complejos K hu- ondas en dientes
frontal (8-12 Hz) occipital < 50% > 50%
sos del sueo de sierra

EMG Activo Descenso Disminuido Atona


Movimientos Movimientos lentos rotatorios
EOG Ausentes Rpidos, conjugados
rpidos en balancn
Profundidad Supercial Media Sueo "profundo" Media
Inestabilidad,
FC, TA, F.resp Descenso, estabilidad
arritmias, apneas

Regulacin PRL (+)


hormonal T GH (+), TSH (-), ACTH (-)
Fenmenos
fsicos
T Homeotermia Poiquilotermia
Parasomnias Bruxismo Sonambulismo, terror nocturno Pesadilla

Pregunta 73. Fases del sueo y fenmenos siolgicos y patolgicos asociados.

que subyacen a esa preocupacin por la alimentacin). con retraso mental de grado moderado en el 75% de los casos y con
convulsiones en el 25%.
Pregunta 73.-R: 4 Trastornos anes al autismo son el sndrome de Asperger (una especia
Vase la tabla correspondiente a esta pregunta. de autismo benigno en el que se conserva el lenguaje y no hay retraso
mental) y el trastorno desintegrativo (una forma de autismo tardo que
Pregunta 74.-R: 4 debuta tras dos aos de desarrollo normal).
La narcolepsia (sndrome de Gelineau) es una enfermedad poco frecuente El sndrome de Rett es un trastorno generalizado del desarrollo que
en la que se combinan: no forma parte del espectro autista; afecta mayoritariamente a nias y
Ataques incoercibles del sueo (tpicamente en sueo REM). aparece tras varios meses de desarrollo psicomotor normal; junto con las
Con la aparicin de caractersticas del sueo REM en la vigilia: graves alteraciones psicopatolgicas se evidencia una desaceleracin del
- Cataplejia (prdida brusca del tono muscular ante las emociones crecimiento ceflico y otros signos neurolgicos; se produce en el 95%
bruscas). de los casos por una mutacin de novo (no hereditaria) en el gen MECP2,
- Parlisis del sueo (falta de recuperacin del tono muscular tras situado en el brazo largo del cromosoma X.
el despertar).
- Alucinaciones hipnaggicas e hipnopmpicas (aparicin o per- Pregunta 76.-R: 4
sistencia de las imgenes de los sueos en la fase de transicin El uso de psicofrmacos en los trastornos mentales de la infancia y la
entre vigilia y sueo). adolescencia est insuencientemente estudiado, siendo muy pocos los
ensayos clnicos realizados en gran medida por la controversia hacia el uso
En su tratamiento se combinan una serie de medidas higinicas que buscan de frmacos psiquitricos en este grupo de edad, dado que las hiptesis
respetar los perodos naturales de sueo e interrumpir la actividad, con dominantes atribuyen la mayora de los trastornos mentales de la infan-
siestas programadas, para disminuir la frecuencia de ataques de sueo con cia y la adolescencia a conictos psicolgicos o problemas relacionales.
diversos medicamentos destinados a aliviar la somnolencia (estimulantes Sin embargo, en algunas entidades est claramente establecida la utilidad
anfetamnicos, modanil) o los fenmenos REM (antidepresivos). de la medicacin, sin que por ello se descarten los abordajes psicolgicos
Tiene un fuerte componente hereditario, asocindose al HLA-DQB1 en o sociales:
un porcentaje muy alto de los casos. Trastorno por dcit de atencin con hiperactividad:
- De primera eleccin: estimulantes anfetamnicos (metilfenidato
Pregunta 75.-R: 2 en Espaa).
El autismo infantil debuta clnicamente antes de los 3 aos de edad, - De segunda eleccin: antidepresivos (tricclicos, atomoxetina);
siendo posible detectar manifestaciones al poco tiempo del nacimiento. modanilo.
Lo fundamental es un dcit en la interaccin social, con ausencia de Trastorno de La Tourette:
reciprocidad social o emocional. Es tambin destacable la alteracin de - De primera eleccin: antipsicticos tpicos incisivos (pimocida,
la comunicacin, con retraso en el desarrollo del lenguaje, dicultad para haloperidol).
mantener una conversacin, as como el uso estereotipado del mismo o - De segunda eleccin: antipsicticos atpicos, clonidina.
bien un lenguaje idiosincrsico. La comunicacin no verbal tambin est Enuresis nocturna:
reducida. La conducta es repetitiva, sin juego creativo. Son frecuentes - De primera eleccin: tcnicas de modicacin de conducta (sis-
las estereotipias y los manierismos, manifestando resistencia a cualquier temas de alarma).
cambio del entorno, incluso llegando a episodios de agitacin. Se asocia - De segunda eleccin: desmopresina, imipramina.

CTO Medicina C/Francisco Silvela, 106 28002 - Madrid Tfno. (0034) 91 782 43 30/33/34 E-mail: secretaria@ctomedicina.com www. ctomedicina. 21
Comentarios de Test a distancia 1. vuelta
Psiquiatra
Pregunta 77.-R: 4 pulsividad es frecuente que vayan apareciendo conductas antisociales
El trastorno por dcit de atencin es probablemente el diagnstico (robos, mentiras, peleas), con una clara tendencia al mantenimiento
psiquitrico ms frecuente en la edad infantil. El sntoma fundamental de esos comportamientos y al diagnstico de trastorno antisocial de
es la dicultad para sostener la atencin un tiempo prolongado, lo que la personalidad en la edad adulta. Un porcentaje importante de los pa-
provoca que los nios sean fcilmente distrables por cualquier estmulo y cientes seguirn presentando sntomas en la edad adulta, siendo muy
se muestren sumamente inquietos. Suele debutar en la primera infancia, elevada su comorbilidad con otros trastornos psiquitricos, sobre todo
pero se hace ms evidente al iniciar la escolarizacin. De sus tres esferas los relacionados con el abuso de sustancias.
sintomticas (dcit de atencin, hiperactividad, impulsividad), las dos Conviene tener en cuenta este diagnstico cuando nos enfrentamos
primeras suelen mejorar rpidamente con el tratamiento farmacolgico, a un nio con problemas de comportamiento en el medio escolar, si
que siempre debe combinarse con una intervencin psicolgica en el bien es necesario que se demuestre la existencia de sntomas de las dos
medio escolar y con la familia; los estimulantes anfetamnicos mejoran la esferas principales (dcit de atencin, hiperactividad), pues existen
capacidad atencional y disminuyen la distraibilidad, permitiendo que el otras muchas causas de mal comportamiento en un nio (trastorno
nio aproveche las horas escolares y se reduzcan los comportamientos negativista-desaante, depresin con predominio de la irritabilidad,
problemticos. Cuando no se diagnostica a tiempo o predomina la im- problemas de adaptacin en pacientes con discapacidad intelectual).

CTO Medicina C/Francisco Silvela, 106 28002 - Madrid Tfno. (0034) 91 782 43 30/33/34 E-mail: secretaria@ctomedicina.com www. ctomedicina.com 22

También podría gustarte